4120 1346
Exam Time
-
NOTEPAD
Results
of 200 questions answered correctly

You have reached of 200 points, ( %)

Your time

Question 1 of 200

1. A 10-year-old boy with symptoms of arthritis and myocarditis was delivered into a hospital. Based on clinical examination the preliminary diagnosis of juvenile rheumatoid arthritis was made. What symptom is the most contributive for the diagnostics of this disease?

Explanation

The question stem clearly states that the patient has been diagnosed with Rheumatoid Arthritis.

So basically, we need to differentiate Rheumatoid Arthritis and Osteoarthritis so that we can arrive at the correct answer specific for Rheumatoid Arthritis.

 

RA: autoimmune which erodes articulated cartilage and bone. The inflammatory cells and cytokines then induce pannus (proliferative granulation tissue) formation, deep usurations.

Associated with pain, swelling and morning stiffness, lasting >1hr and the pain improves with use.

Joint findings: joint space narrowing; erosions; juxtaarticular osteopenia; soft tissue swelling; subchondral cysts

Involves Metacarpophalangeal joints; wrist; proximal interphalangeal joints. Does not affect distal interphalangeal joints on the hands

 

OA: mechanical wear and tear - it’s a degenerative joint disease.

Associated with Pain in weight bearing joints after use (i.e. at the end of the day), and the pain improves with rest.

Joint findings: joint space narrowing; osteophytes (bone spurs); subchondral sclerosis and cysts.

 

Involves distal interphalangeal joints on the hands (heberden nodes) and Proximal interphalangeal joints (Bouchard nodes); does not affect Metacarpophalangeal joints.

 
2. A 25-year-old patient was delivered to an infectious diseases unit on the 3rd day of illness with complaints of headache, pain in lumbar spine and gastrocnemius muscles, high fever, chill. Objectively: condition of moderate severity. Scleras are icteric. Pharynx is hyperemic. Tongue is dry with dry brown coating. Abdomen is distended. Liver is enlarged by 2 cm. Spleen is not enlarged. Palpation of muscles, especially gastrocnemius muscles, is painful. Urine is dark in colour. Stool is normal in colour. The most likely diagnosis is:

Explanation

Leptospirosis is an acute generalized infectious disease, characterized by extensive vasculitis, caused by spirochetes of the genus Leptospira. It presents with flu-like symptoms, myalgias ( majorly calf muscles eg gastrocnemius ), jaundice. Photophobia, icteric scleras etc. The icterohemorrhagic form of leptospirosis presents with a severe form of jaundice and azotemia from the liver and kidney dysfunction coupled with anemia.

 

N/B Virus hepatitis has a gradual onset, without chills, the temperature rises at the pre-icteric period. Muscle pains, scleritis, conjunctivitis are not characteristic of it. There are no meningeal and renal syndromes.

3. For 4 days a 35-year-old man has been treated in the resuscitation unit for acute renal failure caused by compartment syndrome. The patient is disoriented. ECG shows high T waves and right ventricular extrasystoles. His central venous pressure is 159 mmH2O; for the last 3 hours auscultation has been detecting isolated moist crackles in the lungs. Respirations are 32/min. Blood test: residual nitrogen - 62 mmol/L, K+ - 7.1 mmol/L, Cl− - 78 mmol/L, N a+ - 120 mmol/L, Ht- 0.32 L/L, Hb- 100 g/L, blood creatinine - 0.9 mmol/L. In this case the most advisable would be to perform:

Explanation

Hemodialysis is a procedure used for removal of waste materials from the blood through filtration. It is carried out on people with renal failure. Notice that his blood values are not normal; K- 7.1mmol/L ( norm- 3.5-5.5), Cl- 78mmol/L (95-105), Na- 120mmol/L (136-145). Recall that the kidney has three basic functions; filtration, reabsorption and secretion, and a distortion in these values indicates a kidney related issue
4. A 55-year-old woman, a cook, complains of pain in her right knee joint that has been troubling her for a month and intensifi- es in the evening. Objectively she is overweight, the knee joint is swollen, creaks during movement, palpation reveals localized pain. The 1st metatarsophalangeal articulation is deformed on the both feet. No changes in blood and urine are detected. What should be visible on the patient’s X-ray?

Explanation

The patient in question has a case of osteoarthritis or rheumatoid arthritis indicated by joint pain, swelling and stiffness, loss of joint function and deformities. Apart from biochemical analysis, x-ray can also be used for diagnosis; in the x-ray presentation, we observe the narrowing of joint space and presence of marginal osteophytes ( small bony projections found within joint capsules).
5. A 52-year-old woman has been suffering for 2 years from dull, occasionally exacerbating pain in her right subcostal area, occurring after eating high-fat foods, bitter taste in her mouth in the morning, constipations, and flatulence. Objectively she has excess weight, her body temperature is 36.9oC; there is a coating on the root of her tongue; the abdomen is moderately distended and painful in the area of gallbladder projection. What examination would be the most helpful for diagnosis-making?

Explanation

Realise that the patient presents with symptoms indicating a gallbladder related issue ( cholecystitis, cholangitis etc); these symptoms include: pain in the right subcostal area especially after eating food rich in fat, bitter taste in her mouth in the morning, constipations, and flatulence. 

 

An ultrasound allows doctors to view images of the organs and soft tissues inside your body. This procedure is used in diagnosing conditions related to the gallbladder.

6. A 57-year-old woman complains of weakness, dyspnea, loss of appetite, and liquid feces. She has been suffering from this condition for 2 years. Objectively she presents with pale skin, subicteric sclera, and brightred fissured tongue. Lymph nodes are not enlarged. Pulse - 100/min. BP- 105/70 mm Hg. Liver +3 cm, the spleen cannot be palpated. Blood test: erythrocytes - 1.2 · 1012/L, Нb- 56 g/L, color index - 1.4, macrocytes, leukocytes - 2, 5 · 109/L, eosinophils - 1%, juvenile - 1%, metamyelocytes - 1%, band neutrophils - 8%, segmented neutrophils - 47%, lymphocytes - 38%, monocytes - 4%, reticulocytes - 0.1%, platelets - 100 · 109/L, ESR- 30 mm/hour, indirect bilirubin - 26 mmol/L. What changes can be expected in the bone marrow puncture material?

Explanation

7. A 45-year-old woman complains of paroxysmal intolerable facial pain on the left with attacks that last for 1-2 minutes. Attacks are provoked by chewing. The disease onset was two month ago after overexposure to cold. Objectively: pain at the exit points of the trigeminal nerve on the left. Touching near the wing of nose on the left induces new pain attack with tonic spasm of the facial muscles. What is the most likely diagnosis among those listed?

Explanation

Trigeminal Neuralgia is a condition which produces repetitive,unilateral, shooting/shot-like pain in the areas innervated by the trigeminal nerve ( the face). It is triggered by chewing, talking or touching certain parts of the face. Typically lasts for seconds to minutes but episodes often increase in intensity and frequency over time.

Recall that the trigeminal nerve, CV is a mixed type of nerve, has 3 major branches;

 

V1 (ophthalmic), V2( Maxillar), V3(mandibular). This nerve is responsible for mastication, facial sensation, somatosensation from anterior ⅔ of the tongue, and innervates the tensor tympani.

8. The inpatient surgery unit has introduced the method of laparoscopic cholecystectomy into its practice. As the result the average duration of postoperative care provided to the patients could be reduced to 3.4±0.8 days compared to 7.3±1.1 days that were required after non-laparoscopic cholecystectomy. What method of medical statistics can confirm the statistical significance of the difference between these two estimates?

Explanation

9. A 37-year-old worker during a fire ended up in the area of high CO concentration. He was delivered to a hospital in unconscious state. Objectively: the skin of his face and hands is crimson. Respiration rate is 20/min. ECG: alterations specific for hypoxic myocardium. Hourly diuresis is 40 ml. Blood test: erythrocytes - 4.5·1012/L, Нb- 136 g/L, color index - 0.9, ESR3 mm/hour, carboxyhemoglobin - 5%. What criterion allows determining the severity of the patient’s condition?

Explanation

 

The patient who is currently expressing signs of  excess carbon monoxide exposure (unconscious state, discoloration of skin of the hands and face, ecg changes indicating high levels of hypoxia) has normal hemoglobin concentration and color index but a high amount of carboxyhemoglobin ( 2-3% norm).  Recall that hemoglobin binds to oxygen forming oxyhemoglobin; in this form, oxygen is transported to various parts of the body. In the presence of Carbon ( CO₂ or CO), oxygen is preferentially discharged since carbon has a higher affinity for hemoglobin; carbon then binds to hemoglobin (in this case carbon monoxide forming carboxyhemoglobin) thereby reducing the oxygen carrying capacity of hemoglobin. Hypoxia ( reduced oxygen supply) to the brain will lead to a hypoxic coma.

10. The right arm of a newborn is stretched along the torso with all its joints extended; the shoulder is rotated inwards, while the forearm is pronated, the hand is in the position of palmar flexion. Spontaneous movements are absent in the shoulder and elbow joints, passive movements are painless. What is the most likely diagnosis?

Explanation

Duchenne-Erb's palsy is a form of brachial plexus palsy. Erb's palsy leads to a weakness of a newborn baby's arm. It is caused by a stretch injury to the brachial plexus ( C5-C6). The brachial plexus is a network of nerves near the neck that give rise to all the nerves of the arm. These nerves provide movement and feeling to the arm, hand, and fingers (note that - Spontaneous movements are absent in the shoulder and elbow joints, passive movements are painless).

Klumpke paralysis (C7-8, T1) results in weakness of the intrinsic muscles of the hand; grasp reflex is absent. If cervical sympathetic fibers of the first thoracic spinal nerve are involved, Horner syndrome is present.
11. In April during the medical examination of various population groups, 27% of individuals presented with low working ability and rapid fatigability. The following symptoms were observed in the affected individuals: swollen friable gingiva that bleeds when pressed, hyperkeratosis follicularis not accompanied by skin dryness. These symptoms most likely result from the following pathology:

Explanation

 

Vitamin C is needed for the regulation of collagen synthesis (it stabilizes collagen mRNA, thereby increasing collagen synthesis). In its absence, collagen synthesis is unstable leading to reduced wound healing and strength of tissues; symptoms include frequent gingival and petechial hemorrhages as seen in the patient. Vitamin C deficiency is known as Scurvy.

In Vit A deficiency, the patient will experience impaired vision especially at night; this is because Vit A is a precursor of rhodopsin- the photo pigment found in rods (responsible for black and white vision). Vit B1 deficiency leads to Beri Beri

12. A 23-year-old man complains of facial edema, headache, dizziness, low urinary output, urine discoloration (dark red). These complaints arose after the patient had had a case of acute tonsillitis. On examination there are facial edema, the skin is pale, temperature is 37.4oC; heart rate is 86/min., blood pressure is 170/110 mm Hg. Heart sounds are muffled, the II heart sound is accentuated over the aorta. What etiological factor is the most likely in this case?

Explanation

The Patient in question has a rheumatic Fever. RF is gotten from prior infection of the throat ( acute tonsillitis, pharyngitis etc) by group A beta hemolytic streptococci.For diagnosis, the Jones criteria (major criteria)is used. They include migratory polyarthritis, carditis, subcutaneous nodules, erythema marginatum ( rash with ring margin) and sydenham chorea; and some minor criteria such as fever. It is classified as a type II hypersensitivity reaction (immune mediated) and occurs due to molecular mimicry. Aschoff bodies (granuloma with giant cells), anitschkow cells, increased antistreptolysin O and anti-DNase B titers are seen in lab findings.
13. During examination a 4-month-old child with meningococcemia presents with acrocyanosis, cold extremities, tachypnea, and thready pulse, blood pressure of 30/0 mm Hg, anuria, and sopor. What clinical syndrome is it?

Explanation

Septic shock is a type of distributive shock, caused by generalized reaction of the organism to microorganisms and their toxins. Its etiology can be connected with gram positive and gram negative bacteria, rickettsia or fungi and any mucosa can be their entrance (intestine, abdominal cavity, urinary tracts, wound surface). When the toxins get to the blood flow tissue macrophages answer their appearance with production of cytokines. Cytokines are mediators of systemic inflammatory response; they trigger a whole number of immune reactions (both humoral and cellular immune response). This leads to circulation and microcirculation disorders and thus to tissue hypoperfusion and tissue anoxemia.
14. The pregnancy is full term. The body weight of the parturient woman is 62 kg. Fetus is in a longitudinal lie, the head is engaged to the pelvic inlet. Belly circumference is 100 cm. Uterine fundus height is 35 cm. What body weight of the fetus can be expected?

Explanation

 

In recent studies, the value of abdominal girth ( belly circumference) and fundal height ( length from the mother’s uterus to the top of the pubic symphysis) can be used in predicting the weight of a baby. This method is used in order to quickly determine low birth weight babies. It is done by multiplying  the belly circumference by the fundal height; giving you the predicted weight in grams (g), (divide by 1000 for the value in kg). Ie, 100x35= 3500g or 3.5kg.

15. An 18-year-old young man complains of pain in his knee and elbow joints and body temperature up to 39.5oC. One week and a half earlier developed sore throat. On examination his body temperature is 38.5oC. Swelling of the knee and elbow joints is observed. Pulse is 106/min., rhythmic. Blood pressure is 90/60 mm Hg. Cardiac borders are unchanged, heart sounds are weakened, at the cardiac apex there is soft systolic murmur. What factor would be the most indicative of the likely disease etiology?

Explanation

The Patient in question has a rheumatic Fever. RF is gotten from prior infection of the throat ( acute tonsillitis, pharyngitis etc) by group A beta hemolytic streptococci. For diagnosis, the Jones criteria (major criteria)is used. They include migratory polyarthritis, carditis, subcutaneous nodules, erythema marginatum ( rash with ring margin) and sydenham chorea; and some minor criteria such as fever. It is classified as a type II hypersensitivity reaction (immune mediated) and occurs due to molecular mimicry. Aschoff bodies (granuloma with giant cells), anitschkow cells, increased antistreptolysin O and anti-DNase B titers are seen in lab findings.
16. A 20-year-old patient complains of severe headache, double vision, weakness, fever, irritability. Objectively: body temperature is 38.1oC, the patient is reluctant to contact, sensitive to stimuli. There are ptosis of the left eyelid, exotropia, anisocoria S>D, pronounced meningeal syndrome. On lumbar puncture the cerebrospinal fluid flowed out under a pressure of 300 mm Hg, the fluid is clear, slightly opalescent. 24 hours later there appeared fibrin film. Protein - 1.4 g/L, lymphocytes - 600/3 per mm3, sugar - 0.3 mmol/L. What is the provisional diagnosis?

Explanation

Meningitis is an acute infectious disease with involvement of the arachnoid and pia of the brain and spinal cord by pathogenic microorganisms. Etiologically, the various forms include; bacterial, viral. Fungal and tuberculous meningitis. The tuberculous form occurs as a secondary infection; The CSF is under increased pressure, is clear or slightly cloudy, and contains a predominance of mononuclear cells (usually >400/mm3), increased protein (100 to 400 mg/dL), and a decreased glucose content.
17. A 32-year-old woman complains of increasing spastic pains in her lower abdomen that occur after emotional stress. Bowel movements are intermittent: 2-3 defecations after waking in the morning alternate with constipations that last for 1-2 days. Objectively body mass is retained, palpation of the sigmoid colon is moderately painful. Hb- 130 g/L, leukocytes - 5.2 g/L, ESR- 9 mm/hour. Rectoromanoscopy is painful due to spastic condition of the intestine, intestinal mucosa is without changes. Intestinal lumen contains large amounts of mucus. What is the most likely diagnosis in this case?

Explanation

Irritable bowel syndrome is an inflammatory bowel disease characterised by recurrent abdominal cramps (pain), change in form (consistency) and frequency of stool, constipation etc. N/B the patient’s  Bowel movements are intermittent: 2-3 defecations after waking in the morning alternate with constipations that last for 1-2 days ( indicating frequency in stool and constipation). This condition is common in middle aged women and is associated with underlying conditions such as stress, anxiety, depression or a previous case of intestinal infection.

 
  • Crohn’s disease and ulcerative colitis are also inflammatory bowel diseases; Crohn’s disease affects any portion of the GIT while ulcerative colitis affects the colon and presents with a bloody diarrhea.
18. A 1-year-old child with a case of URTI suddenly developed noisy respirations with difficult inspiration, intercostal retractions, and barking cough on the 2nd night after the disease onset. What is the most likely diagnosis?

Explanation

 

The patient presents with symptoms of croup also called laryngotracheobronchitis; classic symptoms include barking cough, noisy breathing (stridor), intercostal retraction (hoover’s sign). This can be caused by a virus ( parainfluenza, influenza A or B) or bacteria (corynebacterium diphtheriae, staph. Aureus, strep pneumonia etc). It is most commonly seen in children aged 6 months- 5years.

19. A 46-year-old woman complains of severe pain attacks in the right lumbar area, which irradiate to the lower abdomen, and nausea. This kind of pain attacks has never been detected in the patient before. Plain abdominal X-ray reveals no pathologic shadows. Ultrasound detects a hyperechogenic mass 1.5 cm in diameter, which reflects sound wave, in the enlarged right renal pelvis. What diagnosis is the most likely?

Explanation

Renal calculus is simply kidney stones. These stones may be composed of calcium, fats, uric acid etc). These stones can cause a blockade in the ureter causing a very severe pain (as seen in  the patient). An x-ray (KUB- kidney, urether, bladder film) can be used for diagnosis. N/B Radiolucent stones cannot be detected by an x-ray; this is why the plain abdominal x-ray did not produce a result in the patient in review. Other diagnostic methods such as ultrasound will be effective.

 

A cyst is a fluid filled cavity. Both cysts and tumors can be seen on x-rays and can be differentiated by the level of contrast on the film.

20. A 60-year-old woman complains of unbearable pain in her right subcostal area. She has a history of acute pancreatitis. Temperature is 38.2oC. Objectively her sclera are icteric. There are no signs of peritoneal irritation. Ortner’s and Gubergrits’ symptoms are positive. Urine diastase is 320 U/L. What is the most likely diagnosis?

Explanation

Ortner’s sign- tenderness on light percussion on right costal margin by the edge of the palm

Gubergrit’s symptom- painful palpation of the abdomen.

 

Chronic pancreatitis is a progressive inflammation of the pancreas with the periods of  exacerbation (acute worsening) and remission with the eventual result of fibrosis and a high risk of malignization (cancer). A positive ortner and Gubergrit’s sign is indicative of pancreas related issues. Notice that the patient’s diastase level is high (norm: 25-125 U/L), diastase refers to groups of enzymes that catalyze the the breakdown of starch to maltose; these enzymes include α,β and γ amylase (especially α). The diastase level is useful in the diagnosing of diseases such as pancreatitis, jaundice, gall stones etc.

21. A 64-year-old man complains of cough with expectoration consisting of blood-streaked mucus, dyspnea, low grade fever and general fatigue. He has been presenting with these symptoms for 3 months. He has been smoking since early adolescence. Objectively to is 37.4oC, respirations are 26/min., pulse is 82/min., rhythmic, blood pressure is 130/85 mm Hg. The right side of the thorax lags behind in the respiratory process, dull percussion sound and acute decrease of breathing activity are observed there. X-ray shows homogeneous shadow of the lung field on the right with mediastinum displacement towards the affected side. What is the most likely diagnosis?

Explanation

Surgically, lung cancer is divided into 3: Central, Peripheral and atypical. Central cancer includes the tumors of the first to third order bronchi ( main bronchi, secondary/lobar bronchi and tertiary/segmented bronchi). Peripheral cancer includes rounded tumour, pneumonia like tumor and pancoast’s cancer. Atypical forms include miliary carcinomatosis etc.

Basic complaints and signs of a central lung cancer include: Constant cough accompanied by hemoptysis, dyspnea, rales and prolonged or obstructive pneumonia.

On physical examination, asymmetry of the chest and delayed breathing movements on the affected side are observed.

On percussion; dull sound on the side of the lesion.

 

On auscultation, bronchial breathing or moist rales are observed

22. For a week a 42-year-old patient has been suffering from fever attacks followed by high temperature, which occur every 48 hours. Body temperature raises up to 40oC and decreases in 3-4 hours with excessive sweating. The patient presents with loss of appetite and general fatigue. The skin is pale and sallow. The liver and spleen are enlarged and dense on palpation. What method of diagnosis verification would be most efficient?

Explanation

This pattern of fever is specific for malaria. The symptom stated above refers to ‘Paroxysm’ ( a cyclical occurrence of sudden coldness followed by shivering and then fever and sweating, occurring every two days (tertian fever) in P. vivax and P. ovale infections, and every three days (quartan fever) for P. malariae. P. falciparum infection can cause recurrent fever every 36–48 hours. For diagnosis, microscopy of blood smear and thick blood film is used (to detect plasmodium).
23. A 16-year-old adolescent living in a rural area has been bitten in the shin by a stray dog. The wound is superficial. Regular vaccination against tetanus was received 3 months ago. What treatment tactics would be the most advisable in this case?

Explanation

Rabies is a viral disease humans get via an infected dog’s bite. For its remedy, the anti rabies vaccine is used. It is a recombinant vaccine.
24. A 26-year-old woman is suspected to suffer from systemic lupus erythematosus with systemic lesions of skin, vessels, joints, serous tunics, and heart tissue that developed after photosensitization. The following is detected in blood analysis: LE cells, antibodies to native ds-DNA, isolated anti-centromere antibodies, rheumatoid factor is 1:100, Wassermann reaction is positive, circulating immune complex is 120 units. What immunological indicators are considered to be specific to this disease?

Explanation

Systemic Lupus erythematosus is an autoimmune disease characterised by appearance of a butterfly rash ( malar rash), arthritis,oral ulcers, endocarditis etc.Organ damage is due to type III hypersensitivity ( immune complex) and type II (to a lesser extent). For diagnosis, Antinuclear antibodies are tested for. Subtypes of antinuclear antibodies specific for SLE include: Anti Smith and anti double stranded DNA antibodies (ds-DNA antibodies).
25. A group of 5 had been resting in a forest, they were drinking alcohol and eating canned mushrooms and cured fish. The next day two of them were hospitalized with disturbed vision, swallowing and respiration; the third one presented with acute general weakness and dry mouth. The remaining two were healthy. A tick was detected on the skin of one of the healthy group members. What is the most likely diagnosis?

Explanation

Botulism is an illness caused by Botulinum toxin , a toxin produced by Clostridium Botulinum ( a gram positive, spore forming, anaerobic bacteria). Foodborne botulism is gotten from eating infected food especially canned food. Symptoms include; disturbed vision (diplopia), flaccid paralysis, dysphagia, dyspnea etc. It is treated with Botulinum antitoxin.

Tick borne encephalitis is gotten from an infected tick’s bite. It presents with symptoms affecting the nervous system eg meningitis, encephalitis etc

Mushroom poisoning presents with GIT upset, increased salivation, blurred vision, respiratory failure.
26. The doctor has an appointment with a patient, who 2 days ago developed severe chest pain on the left, general weakness, high temperature, and headache. Objectively along the 4th and 5th intercostal nerves on the left the skin is hyperemic and there are tight clusters of small vesicles filled with clear serous content. What is the most likely diagnosis?

Explanation

A dermatome is an area of the skin supplied by a single spinal nerve eg. C2 supplies the posterior half of the skull.

 

This patient previously had a chicken pox infection, the above case is just a reactivation of the virus which manifests as Herpes Zoster along the body’s dermatomes. N/B herpes zoster (shingles) is the reactivated form of Varicella Zoster. The neurological areas commonly affected are dorsal root, trigeminal ganglia ( V1 branch- which leads to herpes zoster ophthalmicus)

27. A worker of a blowing shop complains of headache, irritability, sight impairment - he sees everything as if through a ”net”. Objectively: hyperemic sclera, thickened cornea, decreased opacity of pupils, visual acuity is 0.8 in the left eye, 0.7 in the right eye. The worker uses no means of personal protection. What diagnosis is the most likely?

Explanation

Cataract is the opacification ( clouding ) of the lens which leads to a decrease in vision ( visual acuity). The visual acuity in adults is 60/60 or 6/6 which equals 1. Note that the patient in view has reduced visual acuity in both eyes and has a net like vision ( cloudy); these signs indicate presence of cataract.

In conjunctivitis, we see red eyes ( inflammation of the transparent membrane- conjunctiva), it doesn't affect visual acuity.

 

Keratitis is an inflammation of the Cornea. Symptoms include; eye redness, photophobia, eye pain coupled with a reduced visual acuity.

28. A 39-year-old man, a battery attendant, suddenly developed weakness, loss of appetite, nonlocalized colicky abdominal pains, and nausea. Objectively his skin is gray; there is pink-gray stripe on his gums; the stomach is soft and sharply painful. Blood test detected erythrocytes with basophilic stippling and anemia. The patient has a history of peptic ulcer disease of the stomach. There is tendency to constipation. What is the most likely provisional diagnosis?

Explanation

29. A 9-month-old infant presents with delayed tooth eruption and fontanel closure, weakness, and excessive sweating. What type of hypovitaminosis is the most likely in this child?

Explanation

The symptom of delayed fontanelle closure and tooth eruption indicates the 9 month old baby has issues in relation to reduced calcium levels ( calcium is responsible for formation/modelling of bones and teeth). In relation to vitamins, vitamin D is responsible for the intestinal absorption of elements like calcium, magnesium etc. In humans, vit D₃- cholecalciferol and D₂- ergocalciferol are the most important. Deficiency of vitamin D will lead to Rickets (in children) and osteoporosis(in adults).

Vitamin C deficiency will lead to scurvy characterised by bleeding gums, petechial bleeding ( small hemorrhages)

 

Vitamin B1 deficiency leads to Beri-Beri, Vit A deficiency results in night blindness.

30. A 10-year-old girl exhibits high level of physical development ( + 3σ), her body length increased by 10 cm within a year (which is double the norm for her age group), the number of permanent teeth corresponds with the age norm (20), the development of her secondary sex characteristics is three years ahead of her age (Ма, Р, Ах, Menarche). Development rate ahead of her biological age can occur due to:

Explanation

Realise that the 10 year old girl experiences an acceleration in both physical development and sexual characteristics; this indicates that she has a high level of the growth hormone ( somatotropin) either due to increased production, increased receptors or decreased clearance. Somatotropin is a peptide hormone produced by the anterior pituitary gland. Hormones and glands make up the endocrine system.
31. An infant has been born at the 41st week of gestation. The pregnancy was complicated with severe gestosis of the second semester. The weight of the baby is 2400 g, the height is 50 cm. Objectively: the skin is flabby, the layer of subcutaneous fat is thin, hypomyotonia, neonatal reflexes are weak. The internal organs are without pathologic changes. This newborn can be assessed as a:

Explanation

Norms;  gestation period- 280 days or 40 weeks up to 42 weeks

              birth weight- 3-3.5kg ( 3000-3500g)

             Body length- 45-55cm

The baby in question underwent a full term gestation period (41 weeks) but is underweight ( 2500g). This underweight is due to complications that occurred during gestation as stated “The pregnancy was complicated with severe gestosis of the second semester”. We can therefore conclude that the baby who had a full term gestation had some intrauterine growth restrictions.

A premature infant is one which is born before 37 weeks of gestation.

 

A post mature infant is one born after 42 weeks of gestation.

32. A patient suffering from infiltrative pulmonary tuberculosis was prescribed streptomycin, rifampicin, isoniazid, pyrazinamide, vitamin C. One month after the beginning of the treatment the patient started complaining of reduced hearing and tinnitus. What drug has such a side effect?

Explanation

Tuberculosis is a disease caused by an acid fast bacteria, Mycobacterium tuberculosis. Streptomycin, Rifampicin, Isoniazid, pyrazinamide and ethambutol are very active drugs chosen for its treatment. The following are lists of side effects associated with these drugs:

Streptomycin- vestibular and auditory ( ototoxicity) dysfunction, non oliguric renal failure 

Rifampicin- red/ orange discoloration of body fluids e.g. tears, urine; flu-like symptoms, hepatitis, diarrhea, thrombocytopenia.

Isoniazid- hepatitis, peripheral neuropathies

Pyrazinamide- asymptomatic hyperuricemia, joint pain, rash

 

Ethambutol- retrobulbar optic neuritis

33. Caries morbidity rate is 89% among residents of a community. It is determined that fluorine content in water is 0.1 mg/L. What preventive measures should be taken?

Explanation

Dental caries is a bacterial disease that begins with demineralization of the outermost dental enamel and progresses, if not halted, can lead to loss of tooth substance and infection of the dental pulp. Fluoride acts in several ways to prevent caries. The principal action is thought to be that fluoride in dental plaque inhibits the initial demineralization of enamel, and then promotes remineralization of early lesions. Fluorine is gotten from drinking water and food materials. Adequate amount of fluorine in water should be from 0.7-1.5.
34. During appointment with the doctor a man complains of painful itching rashes that appeared on his skin under the beard and moustache one year ago, with frequent exacerbations occurring throughout the year. Objectively the skin of the facial hair growth areas is bluish-purple, thickened, with pustules, erosions, and scabs covering its moist surface. The fistulous tracts are surrounded by the area of loose pink-red granulation and discharge pus. The skin resembles mulberry in appearance. Make the diagnosis:

Explanation

Sycosis is a chronic inflammatory disease involving the hair follicles especially the bearded part of the face and marked by papules, pustules, and tubercles perforated by hairs with crusting.

Rosacea is a long-term disease that affects your skin and sometimes your eyes. It causes redness and pimples.Symptoms can include

 
  • Frequent redness of the face, or flushing

  • Small, red lines under the skin

  • acne

  • A swollen nose

  • Thick skin, usually on the forehead, chin, and cheeks

  • Red, dry, itchy eyes and sometimes vision problems

35. A 60-year-old man presents with ischemic heart disease and heart failure of the IV class according to NYHA (New York Heart Association) that manifests as dyspnea at rest. There are moist crackles in the patient’s lungs. Liver +4 cm, lower limbs are swollen. Ejection fraction is 25%. What sign is the most indicative when determining functional class of heart failure according to NYHA?

Explanation

New york heart association classification of heart failure

Class I - No symptoms and no limitation in ordinary physical activity, e.g. shortness of breath when walking, climbing stairs etc.

Class II - Mild symptoms (mild shortness of breath and/or angina) and slight limitation during ordinary activity.

Class III - Marked limitation in activity due to symptoms, even during less-than-ordinary activity, e.g. walking short distances (20—100 m).Comfortable only at rest.

Class IV - Severe limitations. Experiences symptoms even while at rest. Mostly bed bound patients.

 

Note that the 60 year old patient manifested dyspnea even while at rest. This classification is based on the level of discomfort ( tiredness, shortness of breath etc) a person feels in relation to the distance covered.

36. A 35-year-old man complains of rapidly incresing fatigue, palpitations, ”visual snow”, dizziness. He has a history of peptic ulcer of the stomach. Objectively the skin is pale. Vesicular respiration is observed in the lungs. Systolic murmur is detected over the cardiac apex, heart rate is 100/min., BP is 100/70 mm Hg. The epigastrium is slightly tender on palpation. Blood test: erythrocytes - 3.2 · 1012/L, Нb- 100 g/L, color index - 0.94. What type of anemia is the most likely present in this case?

Explanation

Posthemorrhagic anemia is an anemia which develops as a result of hemorrhage. There are two types of anemias of this group according to the character of hemorrhage: 1) acute posthemorrhagic and 2) chronic posthemorrhagic anemia.

Acute posthemorrhagic anemia arises after fast massive hemorrhage from vessels due to injury or a pathological process.

Chronic posthemorrhagic anemia develops after repeated hemorrhages, (dysmenorrhea, ulcer of  stomach, hemorrhoids etc.)

Sideroblastic anemia is seen in individuals with high amounts of iron, but with the inability to convert these iron contents into hemoglobin.

Iron deficiency anemia is observed in individuals with issues regarding diet, iron absorption, etc. in these individuals, hemoglobin and colour index levels are low.

Hypoplastic  (аplastic) anemia is characterized by oppression hematopoietic functions of red bone marrow and shown by insufficient formation of erythrocytes, granulocytes and thrombocytes or only erythrocytes.

Anemias which arise after destruction (hemolysis) of erythrocytes are called hemolytic.

 
37. A 35-year-old patient has been suffering from an illness for 3 days. 5 days ago he returned from a trip to Africa. The onset of disease was accompanied by fever up to 40oC, chills, acute headache, myalgia. In the axillary region the lymph node enlarged up to 3x6 cm can be palpated. The lymph node is dense, intensely painful, slightly mobile, without clear margins; the skin over the node is hyperemic and tight. Tachycardia is present. Make the preliminary diagnosis:

Explanation

A plague is a bacterial disease spread from one rodent to another by flea parasites and to humans by an infested flea; caused by Yersinia pestis possible signs include severe intoxication, fever, affection of lymphatic system and lungs. Various forms include the Bubonic, septicemic and pneumonic plague; the bubonic form is characterised by spread via the lymphatic system, severe lymphadenitis. Notice that the patient recently returned from africa, which presents a high probability of being bitten by an infected flea.
38. A 34-year-old man on the 3rd day of ceftriaxone treatment for acute otitis (daily dosage - 2 grams) developed diarrhea occurring 5-6 times per day. Feces are without mucus or blood admixtures. Temperature is 36.6oC. Gregersen reaction (occult blood in feces) is negative. Stool culture detected no pathogenic germs. What is the most likely cause of diarrhea in this case?

Explanation

Antibiotics taken affect both good and bad bacterias. A good number of bacteria make up the normal flora of humans; some of which are opportunistic bacterias and some keep these opportunistic bacterias in check. If antibiotics taken affect these good bacterias, the opportunistic ones will cause a negative effect on the body which in most cases presents a diarrheal symptom. An example of these opportunistic bacteria is Clostridium difficile. Dysbiosis refers to the imbalance in the normal flora in the body. Can be associated with disorders such as inflammatory diseases, cancer, bacterial vaginosis etc.
39. A 24-year-old patient visited a doctor complaining of enlargement of his submaxillary lymph nodes. Objectively: submaxillary, axillary and inguinal lymph nodes are enlarged. Chest X-ray shows: enlarged lymph nodes of mediastinum. Blood test: erythrocytes - 3.4 · 1012/L, Hb- 100 g/L, blood colour index - 0.88, platelets - 190 · 109/L, leucocytes - 7.5 · 109/L, eosinophiles - 8%, band neutrophiles - 2%, segmented neutrophiles - 67%, lymphocytes - 23%, ESR - 22 mm/hour. What test must be prescribed to verify the cause of lymphadenopathy?

Explanation

40. During medical examination a cadet in the naval college was detected to have a painless dense ulcer 1.5x0.5 in size in his perianal area at the 2 o’clock position. The ulcer floor resembles ”old fat”. What is the provisional diagnosis?

Explanation

Syphilis is a sexually transmitted disease caused by the spirochete Treponema Pallidum- a spiral shaped microorganism stained by Romanowsky–Giemsa method. It has three stages namely:

  • Primary

  • Secondary

  • Tertiary

 

        Primary stage is characterised by the presence of Hard Chancre which are firm, painless genital ulcers. Lymphadenitis and Lymphagitis are also seen in this stage.

41. A 7-year-old boy has severe pulmonary mucoviscidosis (cystic fibrosis). He complains of dyspnea and blood expectoration. Objectively he presents with lagging physical development, acrocyanosis, hepatomegaly, drumstick fingers, and nail plates resembling a ”clock face”. Provisional diagnosis of chronic pulmonary heart disease is made. What examination would be the most informative for diagnosis confirmation?

Explanation

A doppler echocardiography is a diagnostic method for heart related issues. It is a combination of an echocardiogram (which uses sound waves to create an image of the heart) and a doppler technology ( which shows the velocity ie, speed and direction of blood flow in cardiac tissues). An Electrocardiography shows the electrical activity of the heart.
42. A 10-year-old boy, who was outdoors in windy and cold weather, developed moderate pain and tingling in his fingers and toes. When he had returned home, his parents noticed that the tips of his fingers and toes were white and their sensitivity was lost. The affected areas are warming up, the fingers are tingling and in pain. Skin pallor changed into redness, tingling stopped, slight itching and swelling of the fingers appeared. Determine the frostbite degree in this child:

Explanation

Frostbite is an injury of the skin and underlying tissues that occur due to exposure to cold ( low temperature). There are 4 degrees of frostbite:

I Degree: lasts for about 5-7 days; after warming, paleness changes to hyperemia (redness). Edema of tissues progresses for about 2 days and then it decreases to 6-7 days when shelling (peeling) of epidermis appears. Tactile and pain sensitiveness (sensation) are preserved but sometimes with disorders.  Pain in injured areas could be severe, itching also could be.

 II Degree: characterised by the spreading of edema, appearance of bullaes. The bottom of the opened bullaes are covered with fibrin. Cyanotic skin and difficulty of movement of phalanges. Necrosis of keratic and granular layers. N/B growth layer of the skin is not lost and regeneration appears after 2 weeks. Scars are not formed.

III Degree: Necrosis of all skin layers or even fatty tissue appears. Inflammation develops: firstly aseptic and then on 5th- 7th day purulent . Bullas contain blood . Decrease of tactile and temperature sensation. Edema of tissues spreads on the proximal areas. Firstly skin has cyanotic color then dark brown and black crusts are formed.

 

IV Degree: Necrosis of all skin, fat tissue and even bones and joints. Results in amputation of the affected area.

43. A 25-year-old woman complains of fatigue, dizziness, hemorrhagic rashes on the skin. She has been presenting with these signs for a month. Blood test: erythrocytes - 1.0 · 1012/L, Hb- 37 g/L, colour index - 1.1, leukocytes - 1.2 · 109/L, platelets - 42 · 109/L. What analysis would be the most advisable for diagnosis making in this case?

Explanation

 

Normal Rbc level in the blood ranges from about Male: 4.3 − 5.9 · 1012/L Female: 3.5 − 5.5 · 1012/L and hemoglobin levelMale: 135-175 g/L Female: 120-160 g/L. A reduction in these values in the blood test indicates Anemia. Red blood cells are produced in the bone marrow of large bones and a bone marrow biopsy is needed for diagnosis.

44. A 12-year-old boy with hypertrophic cardiomyopathy complains of dyspnea caused by the slightest physical exertion. Echocardiography detected asymmetrical left ventricular hypertrophy, signs of pulmonary hypertension, and left ventricular dilatation, its ejection fraction is 59%. These developments are indicative of:

Explanation

Ejection fraction refers to how much blood the heart ( left ventricle) pumps at a single contraction.  It ranges from 55%-75%. Heart failure is observed when the heart is unable to meet up with the body's blood and oxygen demand. This is either due to inadequate contraction of the heart (weak heart muscles) or not enough filling of the heart during relaxation majorly as a result of stiffness of the walls.

Heart failure due to weak heart muscles ( reduced contractility power) is referred to heart failure with reduced ejection fraction.

 

Heart failure due to inadequate filling of the ventricle during relaxation but with normal heart contraction is referred to as a Heart failure with preserved ejection fraction.

45. A 32-year-old welder complains of weakness and fever. His illness initially presented as tonsillitis one month earlier. On examination: temperature - 38.9oC, respirations - 24/min., pulse - 100/min., blood pressure - 100/70 mm Hg, hemorrhages on the legs, enlargement of the lymph nodes. Complete blood count shows Hb- 70 g/L, RBC- 2.2 · 1012/L, WBC- 3.0 · 109/L with 32% of blasts, 1% of eosinophiles, 3% of bands, 36% of segments, 20% of lymphocytes, and 8% of monocytes, ESR- 47 mm/hour. What is the cause of anemia in this case?

Explanation

Blast cells are immature WBC. Presence of 32% of Blast cells in the analysis indicates an acute form of leukemia. Less than 10% of blast cells will indicate a chronic form. This patient also has an anemia (post hemorrhagic form) indicated by the low level of RBC. N/B, when there is leukemia, most of the immature cells will be recruited to produce the cancer cells which will cause a reduction in most of the other cell types especially RBC’s.

 

Megaloblastic anemia is seen in vitamin B12 deficiency while chronic hemolytic anemia will show reticulocytes ( immature RBCs) on blood film unlike leukemia that show immature wbc.

46. After a 5-day-long celebration of his daughter’s wedding a 65-year-old patient ”saw” in his yard many cats, chickens, and rats. He tried to chase them away, but was scared off when the animals started scolding him and tried to harm him. What is the likely diagnosis?

Explanation

 

Delirium tremens also known as Alcohol withdrawal delirium is the most severe form of alcohol withdrawal, it is characterised by an altered mental status and hyperactivity of the autonomic nervous system. Remember from the question stem, the patient attended a 5-day long celebration; most likely he has been stressed within these days and stayed without alcohol consumption leading to delirium tremens. Senile psychosis is  brain degenration due to old age.

47. A regional cardiologist is given a task to develop a plan for preventive measures aimed at decreasing cardiovascular mortality rates. What measures should be planned for secondary prevention?

Explanation

Primary prevention of a disease is aimed at preventing the onset (start of a disease) ie, the disease is yet to occur so we aim at preventing it from happening via optimization of lifestyle and living conditions etc.

Secondary prevention is aimed at preventing the progress of a disease ( early diagnosis and treatment) ie, the patient already has the disease, so we are more concerned on stopping it from progressing and preventing its complications.

 
48. During physical exertion a man suddenly developed acute chest pain on the right and dyspnea. Objectively he assumes forced halfsitting position in the bed, presents with diffuse cyanosis, resting tachypnea of 38/min., the right side of the thorax is enlarged and does not participate in the respiratory process; percussion on the right reveals tympanic resonance and absence of respiration.What is the most likely diagnosis in this case?

Explanation

 Pneumothorax is air in the pleural cavity ie, accumulation of air between the lungs and chest wall. A spontaneous pneumothorax is a form of pneumothorax without any underlying lung issue (just happens suddenly). N/B Tympanic resonance sound during  chest percussion indicates the presence of air. Accumulation of air will prevent chest excursion on the affected side during respiration, that is why we see no chest movement on that side

 

Hemothorax, hydrothorax will give dull sounds during percussion because of the presence of fluid.

49. A patient with Morgagni-Adams-Stokes syndrome has fainted when walking up the stairs. The skin is pale, the pupils are dilated, tonoclonic spasms are observed, rib cage is immobile. Make the diagnosis:

Explanation

Preagony, terminal pause, Agony, clinical and biological death all refer to terminal states of the body. 

  •  Preagony- central nervous system is depressed (coma is possible);

heart sounds are weak, pulse is thready, systolic blood pressure is

lower than the critical level of 70 mm Hg; external respiration is

weak and not effective, tidal volume and respiratory frequency are

inadequate; functions of parenchymal organs are violated. Preagony

can last for minutes, hours or even days.

 
  • Terminal pause- patient faints, blood pressure and pulse become hard to measure, respiratory arrest appears and reflexes are lost. Lasts for minutes.

  • Agony- Muscular tone, reflexes and external respiration (chaotic, with auxiliary respiratory muscles participation) are restored.

  • Clinical death- absence of pulse, dilated pupil and lack of photoreaction, gray or cyanotic skin colour ( skin is pale), unconsciousness, lack of reflexes and muscular atony. No respiration (ribcage is immobile).

  • Biological death- Irreversible changes in the body especially the CNS.

50. A 28-year-old woman complains of skin hemorrhages after minor traumas and spontaneous appearance of hemorrhages on the front of her torso and extremities. On examination: the skin is variegated (old and new hemorrhages), bleeding gums. Blood platelets - 20 · 109/L; in the bone marrow there is increased number of megakaryocytes and no platelet production. Treatment with steroid hormones was effective. What is the likely diagnosis?

Explanation

idiopathic thrombocytopenic purpura is also known as Immune thrombocytopenic purpura.It is a clinical syndrome in which a decreased number of circulating platelets (thrombocytopenia) present as a bleeding tendency, easy bruising (purpura), or extravasation of blood from capillaries into skin and mucous membranes (petechiae). The patient in review presents with thrombocytopenia ( low platelet levels );- 20 x 10^9/L ( norm = 150 − 400 · 10^9/L ) and also bleeding gums.

In Hemophilia, patients will present with bleeding from joints (heathroses), easy bruising or a case of bleeding after surgery, dental procedure. Has three forms; Hemophilia A ( Factor VIII deficient), Hemophilia B ( factor IX deficient), C ( factor XI def.).

Osler weber syndrome is a genetic condition characterized by abnormal vessel formation in the skin and mucous membrane of organs eg lungs, brain etc

 

disseminated coagulation syndrome presents with; widespread clotting factor activation, deficiencies in clotting factors leading to bleeding state.

51. A 7-year-old boy has been an inpatient for 1.5 months. He had been delivered to the hospital with complaints of edemas all over his body, low urine output, and headache. Clinical urinalysis: proteins - 7.1 g/L, leukocytes - 1-2 in the vision field, erythrocytes - 3-4 in the vision fi- eld. During the course of treatment the edemas gradually dissipated, headache abated, diuresis normalized. Daily urine proteins - 3 g/L. Biochemical blood test: total protein - 43.2 g/L, urea - 5.2 mmol/L, cholesterol - 9.2 mmol/L. What glomerulonephritis syndrome is the most likely to be present in the patient?

Explanation

Nephrotic Syndrome - Massive proteinuria (> 3.5g/ day) with hypoalbuminemia, edema (due to decrease in oncotic pressure). Occurs due to podocyte damage, focal segmental glomerulosclerosis, membranous nephropathy, amyloidosis or diabetic glomerulonephropathy.

Nephritic Syndrome- usually due to glomerular inflammation which further damages the glomerular basement membrane, loss of rbc into urine leading to hematuria. Occurs in acute post streptococcal glomerulonephritis, rapidly progressive glomerulonephritis, IgA nephropathy, Alport syndrome, membranoproliferative glomerulonephritis. 

 

Observe that the patient loses about 7.1g of protein per litre of urine daily  (> 3.5g/ day) and also presents with edema which indicates a nephrotic syndrome.

52. The mother of a 3-month-old child came to a family doctor with complaints of her child being physically underdeveloped and suffering from cough attacks and dyspnea. Anamnesis: the child is the result of the second full-term pregnancy with the risk of miscarriage (the first child died of pulmonary pathology at the age of 4 months, according to the mother). Body mass at birth is 2500 g. Cough attacks were observed from the first days of life, twice the child was treated for bronchitis. Considering the severity of the child’s condition the doctor made the referral for hospitalization. What diagnosis was most likely stated in the referral?

Explanation

Cystic fibrosis (Mucoviscidosis) is an inherited disorder that results in a buildup of thick and sticky mucus in the lungs, airways, and other organs. Excess mucus in the lungs can lead to coughing, breathing problems, scarring (fibrosis), and an increased risk of lung infections. There is no cure, but treatments can improve both the length and quality of life for people with the disease.

Cystic fibrosis is caused by mutations in the gene that encodes the cystic fibrosis transmembrane conductance regulator (CFTR). This protein functions as a channel across the membrane of cells that produce mucus, sweat, saliva, tears, and digestive enzymes. 

From the history, the mother already had a child that died of a pulmonary pathology (inherited disorder) and the recurrent infections (bronchitis) all point to Cystic Fibrosis.

 

All other options given are not hereditary.

53. A 10-year-old girl complains of stomachache that appears and intensifies after she eats rough or spicy food, sour eructation, heartburn, frequent constipations, headaches, irritability. She has been presenting with these signs for 12 months. Her meals are irregular and consist of dry food. Objectively her diet is suffi- cient in calories. The tongue is moist with white coating near the root. The abdomen is soft and painful in the epigastrium. What method would be optimal for diagnosis-making in this case?

Explanation

The patient in question presents with symptoms related to the gastrointestinal system; heartburn, frequent constipations, headaches, irritability, stomach ache. Diseases such as Gastritis, GERD, Ulcers should all be speculated. To confirm one's diagnosis, an Esophagogastroduodenoscopy should be performed. This is  a procedure during which a small flexible endoscope is introduced through the mouth (or, with smaller-caliber endoscopes, through the nose) and advanced through the pharynx, esophagus, stomach, and duodenum.
54. A 23-year-old patient had taken 1 g of aspirin to treat acute respiratory infection. After that he developed an asthmatic fit with labored expiration that was arrested by introduction of aminophylline. The patient’s medical history is not burdened with allergies. The patient has undergone two surgeries for nasal polyposis in the past. What diagnosis is most likely?

Explanation

Note that the patient has no history of allergy, no infection or strenuous activity were mentioned; these rule out options of Atopy, infectious and exercise induced bronchial asthma respectively. Aspirin induced asthma presents with three Keys; An Asthmatic reaction, intake of aspirin  or a NSAIDs and presence of nasal polyps (all of which are present in the above case). The disorder is thought to be caused by an anomaly in the arachidonic acid metabolizing cascade. When medications such as NSAIDs or aspirin block the COX-1 enzyme, production of thromboxane and some anti-inflammatory prostaglandins is decreased, and in patients with aspirin-induced asthma, this results in the overproduction of pro-inflammatory leukotrienes ( especially cysteinyl leukotrienes), which can cause severe exacerbations of asthma and allergy-like symptoms.
55. A 50-year-old patient was delivered to a hospital with complaints of blood traces in urine. Urination is painless and undisturbed. Macrohematuria had been observed for 3 days. Objectively: kidneys cannot be palpated, suprapubic area is without alterations, external genitalia are non-pathologic. On rectal investigation: prostate is not enlarged, painless, has normal structure. Cystoscopy revealed no alterations. What is the most likely diagnosis?

Explanation

Renal Cell Carcinoma is most common in men 50-70 years old. It manifests clinically with the classic triad of flank pain, hematuria, and flank mass. There could also be secondary polycythemia; fever and weight loss.

From the question stem: 

Supra public area is without alteration and no abnormal findings on cystoscopy (i.e. no problem with the urinary bladder).

External genitalia and prostate - no abnormal findings.

 

So the best option here associated with hematuria (blood traces in urine) is Renal carcinoma.

56. A 36-year-old man has been complaining of marked weakness, low appetite, and nausea for the last year, year and a half. Within the last year he has lost 10 kg of body mass. The skin is darkened, especially on his face, neck, and arms. Skin folds and nipples are hyperpigmented, there are pigment spots on the patient’s inner thighs. Pulse is 60/min., blood pressure is 80/50 mm Hg. What is the provisional diagnosis?

Explanation

Hyperpigmentation, low blood pressure, loss of weight and appetite are all symptoms that indicate Addisson’s disease also called Primary Adrenal Insufficiency. This is an endocrine related pathology in which the adrenal cortex does not produce enough cortisol leading to the above stated symptoms.

 

In Diabetes Mellitus, the patient will present with a high blood glucose level coupled with polyuria, polydipsia and polyphagia.

57. A 5-year-old child that contacts with viral hepatitis in the kindergarten presents with increased body temperature up to 38oC, weakness, low appetite, single case of vomiting, dull pain in the subcostal area on the right. The child is provisionally diagnosed with viral hepatitis. What examination would be the most informative for diagnosis confirmation?

Explanation

Alanine Transaminase (ALT) is a transaminase enzyme primarily found in the liver and kidneys. An Elevation in ALT (liver function test) levels is used in diagnosing liver pathologies such as cirrhosis, hepatitis, liver cancer etc.

 
58. A patient with trauma of the lower third of the forearm volar surface caused by a glass shard came to a first-aid center. Objectively: flexion of the IV and V fingers is impaired, sensitivity of the inner dorsal and palmar surfaces of the hand and IV finger is decreased. What nerve is damaged?

Explanation

Ulnar Nerve: Sensory innervation to the Fifth and medial half of the fourth digits; Motor innervation: flexion, adduction and abduction of 4th and 5th digits. Injury to this nerve causes Claw hand 

Median Nerve: innervates the skin of the palmar side of the index finger, thumb, middle finger, and half the ring finger, and the nail bed; motor innervation: flexion of lateral fingers, thumb opposition, lumbricals of index and middle fingers. Injured in Carpal Tunnel syndrome and leads to ape’s hand.

 

Radial Nerve: Sensory innervation to the dorsum (back) of the hand except the little finger (supplied  by ulnar). Motor innervation to the dorsum of the hand. Injury leads to loss of grip strength, wrist drop.

59. A man diagnosed with closed-angle glaucoma, grade IIa, of the right eye is registered for regular medical check-ups. In the evening an acute glaucoma attack occurred in his right eye; an ambulance was called. What emergency aid would be optimal in this case?

Explanation

 

Glaucoma is a condition characterised by an increase in ocular pressure which gradually damages the optic nerve and leads to loss of vision. Pilocarpine is a cholinomimetic agent indicated for open and closed angle glaucoma. It is also a potent stimulator of sweat, tears and saliva and can be used in treatment of xerostomia (sjogren syndrome). N/B Atropine is contraindicated in Glaucoma as it leads to an increase in intraocular pressure.

60. A 58-year-old patient complains of pain in the lower left extremity, which aggravates during walking, and sensation of cold and numbness in the both feet. The patient has been suffering from this condition for 6 years. Objectively: the skin is pale and dry, with hyperkeratosis. On the left shin hair is scarce. Pulse cannot be detected over the pedal and popliteal arteries and is weakened over the femoral artery. On the right limb pulsation of the popliteal artery is retained. What is the most likely diagnosis?

Explanation

Notice that, there was no pulse detected over the pedal and popliteal arteries. This is a classic sign of ‘atherosclerosis obliterans’ a peripheral artery disease caused by occlusion of a major, medium or small sized artery ( in this case of the lower left extremity).

 

Obliterating endarteritis is an inflammation that takes place in the inner lining of an artery (intima); it is seen in complications in relation to infections such as tuberculous meningitis, syphilis or in severe radiation poisoning.

Raynaud's disease is a rare disorder of the blood vessels, usually in the fingers and toes. It causes the blood vessels to narrow when you are cold or feeling stressed. When this happens, blood can't get to the surface of the skin and the affected areas turn white and blue.

61. A 22-year-old woman, gravida 1, para 0 arrived with complaints of sharply painful contractions that occur every 4-5 minutes and last for 25-30 seconds. Amniotic fluid did not burst. The fetus is in transverse lie, fetal heartbeats are not affected. Contraction ring is acutely painful, located obliquely at the umbilicus. What is the most likely diagnosis?

Explanation

62. During regular check-up the doctor examines a young woman, a student, with height of 162 cm and weight of 59 kg. She complains that in the evening she becomes unable to see clearly the objects around her. Objectively her skin is dry, presents with hyperkeratosis. Her daily ration has the following vitamin content: vitamin A - 0.5 mg, vitamin B1 - 2.0 mg, vitamin B2 - 2.5 mg, vitamin B6 - 2 mg, vitamin C - 70 mg. Make the diagnosis:

Explanation

 

Vitamin A (retinal, retinol, retinoic acid) is a constituent of visual pigment (retinal + opsin= rhodopsin). Rhodopsin is a visual pigment present in Rods ( responsible for dim light or night vision). It is also essential for the normal differentiation of epithelial cells into specialized tissue. Deficiency leads to night blindness, dry scaly skin etc ( all of which are seen in the described patient).

63. An infant is 2.5 months old. The onset of the disease was gradual, the child had normal body temperature but presented with slight cough. Within a week the cough intensified, especially at night; on the 12th day the child developed cough fits occurring up to 20 times per day and followed by vomiting. There was one instance of respiratory arrest. Make the diagnosis:

Explanation

Pertussis (whooping cough) is caused by Bordetella pertussis - a gram negative bacteria, aerobic cocobacillus (rod).

It can be prevented by pertussis vaccine (DPT).

 

The disease usually starts with cold-like symptoms and maybe a mild cough or fever. Early symptoms can last for 1 to 2 weeks.

After 1 to 2 weeks and as the disease progresses, the traditional symptoms of pertussis may appear and include:

* Paroxysms (fits) of many, rapid coughs followed by a high-pitched “whoop” sound

* Vomiting (throwing up) during or after coughing fits

* Exhaustion (very tired) after coughing fits

 

Child coughs on expiration and “whoops” on inspiration. Pertussis can cause violent and rapid coughing, over and over, until the air is gone from your lungs. When there is no more air in the lungs, you are forced to inhale with a loud “whooping” sound. This extreme coughing can cause you to throw up and be very tired.

 
64. A man works in casting of nonferrous metals and alloys for 12 years. In the air of working area there was registered high content of heavy metals, carbon monoxide, and nitrogen. During periodic health examination the patient presents with asthenovegetative syndrome, sharp pains in the stomach, constipations, pain in the hepatic area. In urine: aminolevulinic acid and coproporphyrin are detected. In blood: reticulocytosis, low hemoglobin level. Such intoxication is caused by:

Explanation

Aminolevulinic acid and coproporphyrin are amino acids necessary for the synthesis of Heme. The excretion of these constituents in urine indicates either an inherited or secondary Porphyria. One of the most frequent causes of the secondary form of this disease is Lead poisoning. An increased degradation of porphyrin will lead to a decrease in Hemoglobin synthesis. Also note that the patient experiences Asthenovegetative syndrome ( physical and nervous fatigue), pain in the hepatic area and stomach; these are classical signs of Lead poisoning.

 

Carbon monoxide poisoning will be characterised by headache, lethargy, dizziness, loss of consciousness etc.

65. Due to introduction of a new treatment method, average duration of therapy in the experimental group was 12.3±0.2 days compared to 15.4±0.4 days in the control group that was treated by the old method. What calculations should be made to estimate the statistical significance of the difference in the results?

Explanation

The T-test is a statistical, hypothetical test used to compare the mean (average) of two related variables. For example comparing the mean of the experimental group to that of the controlled group. It lets you know how significant the differences are.

 

The Z-test also compares the means of two populations although it relies on the Variance ( the square root of the mean deviation).

66. Establishments participating in medical examinations include: medical and preventive treatment facilities, hygiene and preventive treatment facilities, sociomedical expert committees, Ministry of Defence medical committees, Ministry of Domestic Affairs medical committees, forensic medicine agency, etc. Specify what service deals with sociomedical assessment of temporary disability:

Explanation

Temporary disability refers to a disability that occurs for a short time period. The medical and preventive treatment facilities are responsible for the assessment and care of such individuals.
67. A 14-year-old boy presents with moderate bronchial asthma in its exacerbation period. What drug should be prescribed to stop an acute attack of expiratory dyspnea?

Explanation

 

Salbutamol is an antiasthmatic agent that belongs to beta 2- agonist group. beta 2 receptors are found in the bronchioles of lungs and the arteries of skeletal muscles. This drug dilates (opens up) the bronchi (bronchodilation). Cromolyn sodium is a mast cell stabilizer, it prevents the release of Histamine and other inflammatory mediators. Dexamethasone is a glucocorticoid, it has anti inflammatory, immunosuppressive and antipyretic effects.

68. A newborn with gestational age of 31 weeks presents with hypotonia and depressed consciousness. Hematocrit is 35%, general cerebrospinal fluid analysis shows increased content of erythrocytes and protein, and low glucose. These data correspond with the clinical presentation of:

Explanation

 

An increased content of Erythrocytes in CSF findings indicates “ intracranial hemorrhage”. Presence of protein and low glucose in CSF are not specific and can indicate pathologies such as an infarct, meningitis, sepsis etc.  Hypertonia and depressed consciousness are also general signs and can be found in any of the above pathology.

69. A 16-year-old girl has primary amenorrhea, no pubic hair growth, normally developed mammary glands; her genotype is 46 ХY; uterus and vagina are absent. What is your diagnosis?

Explanation

Testicular Feminization syndrome also called Complete androgen insensitivity syndrome is a genetic anomaly in which an XY fetus has a defect in androgen receptors leading to a feminine appearance. Usually presents with an absent uterus and fallopian tubes, a rudimentary vagina, scanty or no pubic hair growth, normal and functioning testes etc Genotype 46XY.

 

 Mayer-Rokitansky-Kuster-Hauser syndrome is also called Mullerian Agenesis. A failure in the development of the mullerian duct which leads to uterus, cervix agenesis and vaginal hypoplasia. Secondary sexual characteristics are normal in such individuals because Ovaries do not develop from the mullerian duct.

70. 6 hours ago the waters of a 30-year-old gravida 1, para 0, burst; her preliminary period was pathologic and lasted for over 2 days; the term of pregnancy is 39 weeks. No labor activity is observed. Fetal head presents above the pelvic inlet. Fetal heartbeats are 142/min., clear and rhytmic. On vaginal examination the uterine cervix is not dilated. What further tactics should the doctor choose?

Explanation

71. Mother of an 8-year-old girl complains that the child is too short and has excessive body weight. Objectively: obesity with fat deposits on the torso and face (round moon-like face), acne, striae on the thighs and lower abdomen, hirsutism. What hormone can cause such symptoms, when in excess?

Explanation

“A round moon-like Face” is a term used to describe an individual with Cushing’s syndrome. Occurs when the body produces an increased amount of cortisol ( a hormone produced from the zona fasciculata of the adrenal cortex). Symptoms include;  abdominal obesity but with thin arms and legs, reddish stretch marks, a round red face, a fat lump between the shoulders ( buffalo hump), weak muscles, weak bones, acne, hirsutism.
72. A 9-year-old girl complains of fever up to 38.5oC, headache, inertness, weakness, loss of appetite, stomachache, and frequent painful urination. Provisional diagnosis of acute pyelonephritis is made. Clinical urine analysis: specific gravity - 1016, no protein, leukocytes - 10-15 in the vision field. What investigation method can verify the diagnosis of urinary tract infection?

Explanation

Pyelonephritis is a Urinary tract infection mostly caused by bacteria and leads to an inflammatory process in the kidneys. Can be caused by E.coli, klebsiella, pseudomonas and is being accompanied by symptoms such as fever, frequent urination and pain during urination etc. In the patient described above, for confirmation of the causative agent, a sample of the patient’s urine is collected; a bacteria specimen is obtained from it ( unknown bacteria). It is then introduced into a culture ( inoculation) where they can grow and multiply. N/B various microorganisms have specific culture mediums.
73. A newborn has Apgar score of 9. When should the infant be put to the breast?

Explanation

 

An apgar score is a test used in assessment of the health of a newborn child. In this test, the skin colour, pulse rate, irritability, muscle tone and respiration are checked and scored individually between 0-2. A score of 7 and above means the baby is normal, 4-6 is fairly low, 3 below refers to a critical condition; immediate actions to restore the child’s health should be taken. For a baby with normal response, he/she should be taken immediately to the mother to be breastfed.

74. During assessment of work conditions at the factory manufacturing mercury thermometers, the content of mercury vapors in the air of the working area is revealed to exceed the maximum concentration limit. Specify the main pathway of human body exposure to mercury:

Explanation

 

Realize that the question indicates “content of mercury vapors in the air of the working area”. Since the mercury is in vaporized form, the route of entry will be through the respiratory organs.

75. In the process of hiring, a prospective employee has undergone preventive medical examination and was declared fit to work in this manufacturing environment. What type of preventive medical examination was it?

Explanation

76. A 47-year-old woman came to the admission room with complaints of general weakness, dizziness, vomiting with blood clots. Condition onset was 3 hours ago. The patient has no preceding illnesses. Blood pressure is 90/60 mm Hg, pulse is 106/min., of poor volume. The abdomen is soft, with mild tenderness in the epigastrium. Blood test: erythrocytes - 2.1·1012/L, Нb- 70 g/L, hematocrit - 28%. What tactics should the doctor on duty choose?

Explanation

77. A 23-year-old woman came the the gynecologist with complaints of blood smears from her genital tracts that have been observed for a long time. Her menstruation has been delayed for 8 weeks. Examination shows the uterine body to be enlarged up to 14 weeks of pregnancy. US detected a vesicular mole. What tactics should the doctor choose?

Explanation

From the question, we can pick out; A delayed menstruation for an 8 week period, an enlarged uterine body- These signs of pregnancy are backed up with an ultrasound. The US presents a Vesicular mole ( grape like mole) also called a Hydatidiform mole or molar pregnancy.  This occurs when a non-viable fertilised egg ( an egg without a maternal nucleus) gets implanted into the walls of the uterus. For treatment, the doctor should evacuate the uterus either through suction  or surgically by uterine curettage ( using a special equipment to scrape out uterine tissue).
78. A 52-year-old patient complains of pain in the right part of her chest, dyspnea, cough with large amounts of foamy sputum emitting foul smell and resembling ”meat slops”. Objectively: the patient’s condition is grave, cyanosis is observed, breathing rate is 31/min., percussion sound above the right lung is shortened, auscultation revealed various moist rales (crackles). What is the most likely diagnosis?

Explanation

Meat slops’ sputum is a specific term  used by Krok to  describe a lung gangrene. A lung gangrene is a diffuse purulent necrosis of the tissue without the tendency of a defined demarcation.

Pleural empyema is a purulent inflammation of its visceral and parietal membranes, which is associated with accumulation of pus in a pleural space.

Lung abscess is defined as necrosis of the pulmonary tissue and formation of cavities containing necrotic debris or fluid caused by microbial infection.
79. A man complains of sore throat on the left, pain in his left ear, to up to 39oC, and nasal sound of his voice. Disease onset was 5 days ago. Marked trismus and increased salivation are observed. The head tilts to the left shoulder. Left side of the soft palate presents with swelling, hyperemia, and infiltration. Retromandibular lymph nodes on the left are acutely painful on palpation. Otoscopy results are normal. Make the diagnosis:

Explanation

 

Extension of a tonsil infection can take place in the surrounding tissue and is referred to as  a Peritonsillar abscess or quinsy. This complication of tonsillitis is usually caused by a B-hemolytic streptococcal infection and are characterised by an extremely sore throat and high fever ( as seen in the patient) . Note that, Retropharyngeal abscess is also a complication of tonsillitis but is virtually limited to infants in the first two years of life; the characteristic findings are fever, hyperextension of the neck, dysphagia and noisy respiration.

80. A patient has the second and third degree burns of the 15% of the body surface. On the 20th day after the trauma the patient presents with sharp increase of body temperature, general weakness, rapid vesicular respiration; facial features are sharpened, BP is 90/50 mm Hg, heart rate is 112/min. What complication is it?

Explanation

Sepsis is a life-threatening organ dysfunction caused by a dysregulated host response to infection. 

It can be clinically assessed when there is Fever (>38*C); Respiratory rate >22cpm; Tachycardia >100bpm; Systolic BP <100mmHg; altered consciousness.

 

Sepsis is a common complication when there is overwhelming infection. In this case, it is a complication of burns injury (presents 20th day after the trauma).

81. A patient in the state of clinical death is being resuscitated through mouth-to-mouth artificial pulmonary ventilation and external cardiac massage. A doctor noticed that air does not flow into the patient’s airways and his head and torso are positioned at the same level. Why is artificial respiration ineffective in the given case?

Explanation

 

During mouth-mouth pulmonary ventilation, the usual thing that blocks the airway is the tongue. This is because, in an unconscious state the tone of the tongue muscles is weak (lacks strength or force); making the tongue fall towards the back of the throat, blocking the airway.

82. An employee has been sick for 4 months, further treatment is necessary, the patient is unable to work. Who is authorized to provide further disability examination of this patient?

Explanation

The right of prolongation a medical certificate of invalidity alongside with the medical treatment by the doctor is granted by:

• Chief medical officer (assistant of the chief medical officer on expertise of ability for work);

• Manager of department;

• Medical-consulting commission also known as medical expertise committee

 

The medical-consulting commission (MCC) is formed in case the doctors staff is above 15 doctors.

83. A population of a small town often presents with registered cases of juvenile cardiomyopathy, atherosclerosis, hypertension, endocrinopathy, chronic dermatitis, and arthralgia - signs of Keshan disease. What is the most likely cause of this pathology?

Explanation

Keshan’s disease is an endemic cardiomyopathy caused by a dietary deficiency of selenium and can be triggered by a mutated strain of the Coxsackie virus; the disease is characterised by heart failure and pulmonary edema and increases the chances of having cancer, hypertension and cardiovascular related issues. It is corrected by administering  Selenium supplements.
84. In 2 hours after a traffic accident a 28-yearold man in grave condition was delivered to a hospital. The patient complains of abdominal pain. He received a blow to the abdomen with the steering wheel. Objective examination revealed the following: the abdomen does not participate in respiration, is tense and acutely painful on palpation; abdominal guarding is present, peritoneal irritation signs are positive, hepatic dullness is absent. BP is 90/60 mm Hg, heart rate is 120/min. What further treatment tactics should be chosen?

Explanation

 Laparotomy: is a surgical incision of the abdominal cavity performed to examine the abdominal organs and aid diagnosis. Indications include: abdominal hemorrhage or hemoperitoneum, blunt chest trauma, peritonitis, intestinal obstruction etc and is contraindicated in cases of severe sepsis and malignancy.

Laparoscopy: A surgical procedure in which the abdominal organs are observed with the use of a laparoscope (a camera). Unlike a laparotomy, it uses small incisions. It is also known as a keyhole surgery or minimal invasive surgery. Indications include: Cholecystectomy, gastric bypass. 

Laparocentesis: is simply the puncture of the peritoneal cavity to obtain its fluid.

 

Realize that the above patient received a blow to the abdomen with the steering wheel- which indicates a blunt trauma; peritoneal irritation signs are positive- and indication for peritonitis.

85. A 48-year-old woman was arrived to the surgical unit with wounds in her thigh. On examination the wound surface has dirty-gray coating with unpleasant sweet smell. Wound content resembles raspberry jelly. Skin tissues around the wound are glossy and turgid. Palpation reveals moderate crepitation in the tissues. What microflora is the most likely to cause such inflammation?

Explanation

Clostridium is a gram positive anaerobic, spore forming bacillus. Its major species include C. Botulinum- botulism; C. perfringens- enterocolitis, gas gangrene; C. Tetani- tetanus; C. difficile- pseudomembranous colitis

 

The description of the wound in the above patient (dirty gray coating, unpleasant sweet smell, raspberry jelly content, moderate crepitation of surrounding tissues upon palpation) indicates a Gangrene. Gangrene is a serious condition where a loss of blood supply causes body tissue to die. Loss of blood supply is most likely due to  an illness, injury or infection ( mostly C. Perfringens).

86. The gynecology unit received a patient with uterine bleeding that started 6 hours after induced abortion at the term of 11-12 weeks. Objectively the skin is pale, pulse is 100/min., blood pressure is 100/70 mm Hg. On vaginal examination the uterus is painless, its enlargement corresponds to the 10th week of pregnancy; uterine cervix is dilated enough to let in one finger, there are fragments of the fertilized ovum. What actions should be taken next:

Explanation

Recall that Curettage is simply using an instrument (curette) to scrape out tissues especially in the uterus.  Indications for this procedure include:

  • Abnormal uterine bleeding: irregular bleeding, menorrhagia, suspected malignant or premalignant condition

  • Retained material in the endometrial cavity

  • Evaluation of intracavitary findings from imaging procedures (abnormal endometrial appearance due to suspected polyps or fibroids)

  • Evaluation and removal of retained fluid from the endometrial cavity (hematometra, pyometra) in conjunction with evaluating the endometrial cavity and relieving cervical stenosis

  • Office endometrial biopsy insufficient for diagnosis or failed due to cervical stenosis

  • Endometrial sampling in conjunction with other procedures (eg, hysteroscopy, laparoscopy).

 

The above patient presented with uterine bleeding and presence of fertilized ovum fragments- these are indications for a Curettage.

87. A woman came to the general practitioner with complaints of fatigability, significant weight loss, weakness, and loss of appetite. She has been presenting with amenorrhea for the last 8 month. One year ago she gave birth to a live full-term child. Blood loss during delivery was 2 liters. The woman received blood transfusion and blood components. What is the most likely diagnosis?

Explanation

 

Notice that the woman in question gave birth about a year ago and experienced a severe blood loss during delivery; this is the leading cause of sheehan’s syndrome. Sheehan’s syndrome is one of the major causes of hypopituitarism in females; it is due to pituitary infarction as a result of postpartum hemorrhage.  fatigability, significant weight loss, weakness, and loss of appetite all include associated symptoms.

Also called Stein- Leventhal syndrome, Ovarian sclerocytosis is the process of ovarian regeneration, accompanied by the formation of small cystic formations up to 1 cm in size. It usually occurs in patients with polycystic ovarian syndrome. Key findings include; infertility, male pattern hair distribution, weight loss, hormonal disbalance, bilateral enlargement of the ovaries, violation of menstrual cycle etc.

Somatoform disorders - multiple, recurrent and frequent somatic complaints requiring medical attention without association with any physical disorder.

88. To assess the effectiveness of medical technologies and determine the power and direction of their effect on the public health indicators, the research was conducted to study the immunization rate of children and measles incidence rate by district. What method of statistical analysis should be applied in this case?

Explanation

89. A 22-year-old woman complains of itching and profuse discharge from her genital tracts. The condition developed 10 days ago after a sexual contact. Bacterioscopy of a discharge sample detected trichomonads. What drug should be prescribed for treatment in this case?

Explanation

“ Bacterioscopy of a discharge sample detected trichomonads”- Trichomonads refer to a group of protozoans that possess about 3-6 flagellas. Examples include Trichomonas vaginalis, buccalis etc. 

Metronidazole is an antiprotozoal and antibiotic drug. It is indicated in several pelvic inflammatory diseases, bacterial vaginosis and protozoal diseases such as giardiasis, trichomoniasis etc. Most suitable treatment for the diseases caused by Trichomonads.

Ampicillin and Erythromycin are antibiotics; they are sometimes used in combination with anthelmintics/ antiprotozoal drugs for treatment of protozoal infections.

 

Acyclovir and valacyclovir are both antiviral drugs and are indicated in diseases such as herpes.

90. A 30-year-old patient was hospitalized in an intensive care unit with a diagnosis of multiple bee stings. Skin is pale and covered with cold sweat. Pulse can be palpated only at the carotid arteries and is 110/min.; breathing rate is 24/min., rhytmical, weakened. What drug must be administered immediately?

Explanation

Recall that the above patient is in the ICU, she presents with symptoms that indicate a state of shock ( in this case an anaphylactic shock due to a bee sting). These symptoms include- an increased pulse and breathing rate, pale skin etc. In cases of a severe outcome from a bee sting, Epinephrine is given. 

Epinephrine Hydrochloridum is a non selective adrenergic agonist i.e., it acts on α₁ α₂ β₁ β₂ β₃ receptors. Its properties include; vasoconstriction, bronchodilation, reduction of Intraocular pressure amongst others. Commonly used in cases of bronchospasm, Anaphylaxis ( as earlier stated) and cardiopulmonary resuscitation.

 

An antihistamine drug such as tavegyl (clemastine) might be considered the correct option if the patient was in a milder and stable state.

91. A 46-year-old woman has been hospitalized with open fracture of the left thigh in its middle third. She underwent the surgery - fixation with extraosseous osteosynthesis plates. On the 4th day after the surgery she developed pain in the wound, body temperature rose over 39oC. What measures should be taken in this case?

Explanation

 

The Presence of an increased body temperature and pain a few days after the wound closure indicates ‘An Infection’ - most likely a bacterial form. To correct this, the sutures/wound dressing should be undone,  drained ( if there be any purulent process), an antiseptic should be used in cleaning the affected area coupled with the administration of an antibiotic therapy to the patient.

92. A woman with blood group B(III) Rh(+) gave birth to a full-term healthy boy. Examination on the 3rd day of the infant’s life shows him to have icteric tint to his skin. The child has no problems with suckling, sleep is nondisturbed. The abdomen is soft, the liver protrudes by 2 cm from under the costal margin. Complete blood count: hemoglobin - 200 g/L, erythrocytes - 5.5 · 1012/L, total bilirubin - 62 mcmol/L, indirect bilirubin - 52 mcmol/L. What condition can be suspected?

Explanation

Due to the inability of the fetal liver to conjugate and quickly excrete the increased amount of bilirubin after birth, a buildup of bilirubin in the blood takes place in the newborn. These accumulated products are deposited in areas like mucous membrane, eyes, skin and in complicated cases- brain. This process is termed ‘Physiological Jaundice’; it occurs within the first 24hrs of birth and resolves without treatment in 1-2 weeks.

Hemolytic disease of the newborn due to Rh incompatibility occurs when an Rh- mother presents with an Rh+ fetus. In the first pregnancy, the mother is exposed to fetal blood and forms a maternal anti-D IgG. In subsequent pregnancies, the anti-D IgG crosses the placenta, attacks fetal RBCs and leads to hemolysis in the fetus.

 

Hemolytic disease of the newborn due to ABO incompatibility occurs when a type O mother presents with a type A or type B  fetus. Pre-existing maternal anti-A or anti-B IgG antibodies cross the placenta leading to hemolysis in the fetus.

93. A 45-year-old man developed constricting retrosternal pain that occurs during walks at the distance of 200 m. Objectively heart rate is 80/min., BP is 160/90 mm Hg. During cardiopulmonary exercise test at 50 W there is a depression of S-T segment by 3 mm below the isoline in V3-V4. What is the provisional diagnosis?

Explanation

An angina refers to chest pain due to an ischemic myocardium secondary to coronary artery narrowing or spasms. Canadian Cardiovascular Society grading of angina pectoris is a classification used to grade the severity of Angina.

Class 0: Asymptomatic Angina- Mild myocardial ischemia with no symptoms.

Class I: Angina only with strenuous exertion- Presence of angina during strenuous, rapid, or prolonged ordinary activity (walking or climbing the stairs).

Class II: Angina with moderate exertion- Slight limitation of ordinary activities when they are performed rapidly, after meals, in cold, in wind, under emotional stress, during the first few hours after waking up, but also walking uphill, climbing more than one flight of ordinary stairs at a normal pace and in normal conditions.

Class III: Angina with mild exertion- Having difficulties walking one or two blocks or climbing one flight of stairs at normal pace and conditions.

Class IV: Angina at rest- No exertion needed to trigger angina.

 

The patient in perspective experiences anginal symptoms after he walks about 200m ( a short distance).

94. A multigravida on the 38th week of her pregnancy complains of increased BP up to 140/90 mm Hg, edema of the shins for 2 weeks. In the last month she gained 3.5 kg of weight. Urine analysis: protein - 0.033 g/L. Make the diagnosis:

Explanation

  1. Preeclampsia, a pregnancy-specific condition in which hypertension develops after 20 weeks of gestation in a previously normotensive woman. It is characterized by hemoconcentration, hypertension, and proteinuria. The following are the differences between a mild and severe Preeclampsia: 

 

Mild Preeclampsia

Severe Preeclampsia

BP reading of 140/90 mm Hg twice, 4-6 hrs apart

Proteinuria of 0.3 g/L in a 24 hr specimen or >0.1 g/L in a random day-time specimen on two or more occasions 6 hr apart

Dependent edema, some puffiness of eyes, face, fingers; pulmonary edema absent

Output matching intake, ≥30 ml/hr or <650 ml/24 hr

Rise to >160/110 mm Hg on two separate occasions 4-6 hr apart with pregnant woman on bed rest

Proteinuria of >0.5 g/L in 24 hr

Generalized edema, noticeable puffiness; eyes, face, fingers; pulmonary edema possibly present

<20 ml/hr or <400 ml to 500 ml/24 hr

 
95. For the last 15 years a 48-year-old patient has been working at the factory producing synthetic resins. Lately he has been complaining of significant general fatigue, headaches, frequent urination (predominantly during the day), red color of urine. What complication of benzene nitrocompounds poisoning can be suspected?

Explanation

Aromatic compounds such as benzene and its products (eg. nitrobenzene, benzidin etc) are major components in factories producing dyes and resins and upon exposure are dangerous  to health. One major negative effect is the conversion of hemoglobin to methemoglobin. In recent times, it has been proved that these compounds act as carcinogens and lead to bladder related tumors.

Chronic Prostatitis is ruled out since the patient wasn’t experiencing pain while passing out urine.

 

In Chronic Cystitis, the patient often observes severe burning sensation in the pelvic region.

96. A 44-year-old patient with postinfarction cardiosclerosis presents with frequent heart rate disorders and lower extremity edema. Objectively: Ps- 95/min., irregular, 10- 12 extrasystoles per minute. BP- 135/90 mm Hg. The 1st heart sound at the apex is weakened. Pulmonary respiration is rough. The liver is enlarged +2 cm. ECG: irregular sinus rhythm, heart rate - 95/min., frequent polytopic ventricular extrasystoles. What antiarrhythmic drug is advisable in this case for treatment and prevention of extrasystole?

Explanation

Realize that the patient experiences about 10-12 extrasystoles per minute, irregular sinus rhythm with a heart rate of 95bpm, frequent polytopic ventricular extrasystoles. Amiodarone is a class III antiarrhythmic drug; a potassium blocker. It prolongs phase 3 of the cardiac action potential (in this phase, calcium permeability is decreased while potassium permeability is increased- it is also known as the repolarization phase). This drug is indicated in Supraventricular tachycardia, ventricular tachycardia, atrial fibrillation etc.

N/B If phase 3 is prolonged, the heart rate slows down.

 

Lidocaine,mexiletine, novocaine and quinidine are all class I drugs ( sodium channel blockers) They basically affect conduction ( QRS complex) and have little or no effect on sinus rhythm.

97. A 60-year-old woman started feeling weakness, vertigo, rapid fatigability during the last year. Recently she has developed dyspnea and paresthesia observed. Objectively: skin and mucous membranes are pale and icteric. Lingual papillae are smoothed out. Liver and spleen are at the edge of costal arch. Blood test: Hb- 70 g/L, erythrocytes - 1.7 · 1012/L, blood color index - 1.2, macrocytes. What drug  can be prescribed on pathogenetic grounds?

Explanation

98. After excessive consumption of fatty food a 60-year-old woman suddenly developed pain in her right subcostal area, nausea, bile vomiting, sharp bitter taste in her mouth. In 2 days she developed jaundice, her urine darkened. Objectively: sclera and skin are icteric, the abdomen is distended, the liver is enlarged by 3 cm, soft and painful on palpation, Ortner’s, Murphy’s, Kehr’s, Zakharyin’s, Mayo-Robson’s signs are positive. What diagnostic technique should be used in the first place to confirm the diagnosis?

Explanation

Ortner’s sign- tenderness on light percussion on right costal margin by the edge of the palm

Murphy’s sign- a delay of breathing during palpation of gallbladder on inspiration

Kehr’s point- acute pain at the tip of the shoulder

Coupled with the positive result from the above stated signs, the patient also  experienced bile vomiting, sharp bitter taste in her mouth, jaundice etc. These are signs specific for pathologies relating to either the gallbladder or liver. Therefore to confirm, the individual should undergo an ultrasound.

 

A fibrogastrodudenoscopy will be the preferred choice of diagnostic method if it was a Gastrointestinal tract related issue.

99. The objective of a statistical research was to find out to what extent the population peruses the available medical services. For this purpose 300 residents of the area were interviewed. Information was collected by means of a special questionnaire. What method of collecting information was used by the researchers?

Explanation

The questionnaire passed across is a way of getting vital information. Anamnesis refers to the collection of previous/ existing information.

 
100. A 30-year-old woman complains of increased body weight and problems with physical exertion. Her parents are of increased body weight as well; typical meals in their family are high in fats and carbohydrates. Objectively her blood pressure is 135/80 mm Hg, pulse is 89/min., weight is 87 kg, height is 165 cm. The patient’s skin is clear, with even distribution of subcutaneous fat; the thyroid gland is not enlarged; there are no menstrual cycle disturbances. What obesity prevention methods would be the most advisable in this case?

Explanation

 

It was outlined that  ‘typical meals in their family are high in fats and carbohydrates’, the first step in avoiding obesity should be to tackle that statement by changing the patient’s  dietary regime.

101. A 43-year-old woman complains of pain in the lumbar area, which irradiates to her left leg and aggravates on movement, and sensation of numbness in this leg. Objectively palpation of her shin and thigh is painful, there are painful stretch symptoms of on the left and gastrocnemius cramps. There is no sensory loss or weakening of reflex responses. Make the diagnosis:

Explanation

Vetebrogenous lumbar ischialgia  is one of the clinical forms of reflex syndromes at lumbar – sacral level. Characterised by pain located in the lumbar area and irradiates towards the hips, legs ( as seen in the patient). It has three forms namely;

  • Muscle-tonic

  • Neurodystrophic

  • Autonomic-vascular

 

In Vertebrogenic radicular syndrome of L5-S1, pain is experienced in the external posterior surface of the  hip, crus, foot, the IV –th and Vth toes; sensory disorders in the same zones, paresis of toe flexors, absent or low achilles tendon reflex

102. A 3-day-old infant with hyperbilirubinemia (428 mcmol/L) developed disturbances manifesting as periodical excitation and convulsions against the background of inertness, hypotension, hypodynamia, and inhibition of unconditioned reflexes, convergent strabismus, rotational nystagmus, and settingsun eye phenomenon. What is the most likely cause of such symptoms?

Explanation

 

Newborns are at a high risk of developing encephalopathy caused by hyperbilirubinemia due to the fact that soon after birth, the infant’s liver gets busy in fetal hemoglobin breakdown (which is later replaced by the adult form) and as a result, the conjugation process of bilirubin  is slowed down leading to an accumulation of unconjugated bilirubin in the blood. Coupled with  a not too developed blood brain barrier, the accumulated bilirubin easily goes across the barrier leading to encephalopathy.

103. A woman undergoing in-patient treatment for viral hepatitis type B developed headache, nausea, recurrent vomiting, memory lapses, flapping tremor of her hands, rapid pulse. Sweet smell from the mouth is detected. Body temperature is 37.6oC, heart rate is 89/min. What complication developed in the patient?

Explanation

Acute liver failure is a state of hepatic cells dysfunction, caused by unknown earlier liver disease, resulting in general intoxication, coagulation violations, neurological and mental disorders. Its etiology is usually connected with: viral hepatitis (hepatitis B virus, hepatitis A virus), poisonings (mushrooms, dichloroethane, phosphorus, carbon tetrachloride, arsenic), eclampsia, burn disease, anaesthetic gas, antibiotics, sulfanilamides, massive bacterial pneumonia, cirrhosis, hepatic tumours and metastasis.

 

Advanced liver failure manifests in a coma (due to the effect of accumulated toxic material on the brain). The patient in question presents with viral hepatitis B ( the probable cause of the liver failure) and a list of other symptoms which indicate neurological and intoxication disorder.

104. A patient with chronic pancreatitis complains of diarrhea occurring up to 5 times per day (no blood traces), loss of body weight, abdominal distention, dryness of skin, loss of hair, thirst, bleeding gums, convulsions. Complete blood count: leukocytes - 5.8 · 109/L; Hb- 86 g/L; ESR- 15 mm/g; Blood protein test: total protein - 48 g/L; albumins - 28 g/L. What indicators of coprological analysis would accompany this syndrom?

Explanation

 

The pancreas plays a major role in digestion ( its exocrine function). It produces digestive enzymes such as; trypsin and chymotrypsinogen ( for protein digestion), Amylase (for carbohydrate digestion), lipase ( for breakdown of fats). In pathologies relating to the pancreas such as pancreatitis, this function is affected leading to a digestion and absorption issue. For example, due to a disruption in the release of lipase, fat breakdown is negatively affected leading to not enough absorption and its appearance in stool ( steatorrhea). Likewise a disruption in chymotrypsin and trypsin secretion will lead to an abnormal excretion of muscle fibres in faeces (creatorrhea).

105. A 43-year-old woman complains of persistent stomachache with recurrent pain attacks, nausea, repeated vomiting with stagnant bowel content, abdominal distension, and flatulence. She has been presenting with these signs for 7 hours. Pulse is 116/min. The tongue is dry and brown. The abdomen is symmetrically distemded, soft, painful. Percussion reveals tympanitis. On auscultation there are bowel sounds with metallic overtone, splashing, and dripping. Make the diagnosis:

Explanation

Intestinal obstruction is a complete or partial violation of passing of contents by the intestinal tract. We observe that the patient experiences repeated vomiting with stagnant bowel content, abdominal distension, and flatulence- these are signs indicating a blockage of free passage of GIT content.

 

N/B the patient does not experience:  pain in the right upper quadrant - this will be indicative of a cholecystitis; An acute pain in the mid abdominal region that radiates to the back- will indicate Pancreatitis

106. After overexposure to cold a 45-year-old woman developed acute pain in her suprapubic and lumbar areas during urination, sharp pains at the end of urination, false urges to urinate. Urine is turbid with blood streaks. The doctor suspects urinary tract infection. What results of laboratory analysis would be the most indicative of such infection?

Explanation

 

When you have a UTI, the lining of the bladder and urethra become red and irritated just as your throat does when you have a cold. The irritation can cause pain in your lower abdomen pelvic area and even lower back, and will usually make you feel like urinating more often. Burning or pain when urinating is the most common symptom. You may even feel a strong urge or need to urinate but only get a few drops. This is because the bladder is so irritated that it makes you feel like you have to urinate, even when you don't have much urine in your bladder. UTIs can be found by analyzing a urine sample. The urine is examined under a microscope for bacteria or white blood cells (leukocyturia) , which are signs of infection. Blood can also be found in the urine sample (hematuria); this can be due to an infective or mechanical cause.

107. A 15-year-old adolescent girl came the the gynecologist with complaints of painful menstruations that are accompanied by nausea, vomiting, and dizziness. Her menarche was at 12. Menstruations became painful since she was 14, remain regular. What treatment should be prescribed in this case?

Explanation

 

Menarche refers to the first menstrual cycle. Painful menstruation (dysmenorrhea) expresses as cramping pain in the abdomen, it is most times associated with symptoms such as lower back pain, nausea, diarrhea, and headaches. These muscle cramps occur as a result of increased prostaglandin levels produced from the uterus. These prostaglandins cause the uterus to continuously tighten and relax, leading to cramps. For treatment, the patient can be given NSAIDs - for pain and anti-inflammatory effect; Antispasmodics- for relaxation of muscles and antiprostaglandin therapy- to reduce the amount and effect of prostaglandins produced.

108. A 38-year-old patient has been delivered by an ambulance to a surgical department with complaints of general weakness, indisposition, black stool. On examination the patient is pale, there are dotted hemorrhages on the skin of his torso and extremities. On digital investigation there are black feces on the glove. Blood test: Hb- 108 g/L, thrombocytopenia. Anamnesis states that similar condition was observed 1 year ago. Make the diagnosis:

Explanation

Immune thrombocytopenic purpura (ITP) is a clinical syndrome in which a decreased number of circulating platelets (thrombocytopenia) present as a bleeding tendency, easy bruising (purpura), or extravasation of blood from capillaries into skin and mucous membranes (petechiae). Recall that the patient presents with dotted hemorrhages on the skin (petechiae) and on analysis, thrombocytopenia. Caused by the binding of an autoantibody (specifically IgG) to platelets leading to platelet destruction.

In Hemophilia, patients will present with bleeding from joints (hemathrosis), easy bruising or a case of bleeding after surgery, dental procedure. Has three forms; Hemophilia A ( Factor VIII deficient), Hemophilia B ( factor IX deficient), C ( factor XI def.).

 
109. A 35-year-old patient developed an epileptic attack with tonoclonic spasms that lasted for 3 minutes. After the attack the patient fell asleep but in 5 minutes the second attack occurred. The first step of emergency aid would be to:

Explanation

  1. If you are with someone having a tonic-clonic seizure (where the body stiffens, followed by general muscle jerking), try to: 

  • Stay calm and remain with the person.

  • If they have food or fluid in their mouth, roll them onto their side immediately.

  • Keep them safe and protect them from injury.

  • Place something soft under their head and loosen any tight clothing.

  • Reassure the person until they recover.

  • Time the seizure, if you can.

  • Gently roll the person onto their side after the jerking stops.

 

Do not put anything into their mouth or restrain or move the person, unless they are in danger. The major step is to ensure the airway is clear, to keep them breathing until the seizure stops.

110. A young woman suffering from seborrhea oleosa has numerous light-brown and white spots on the skin of her torso and shoulders. The spots have clear margins, branny desquamation, no itching. What provisional diagnosis can be made?

Explanation

The above description is typical for Tinea (pityriasis) versicolor which is caused by malassezia spp. A yeast like fungus. It is characterised by areas of hypopigmentation ‘white spots’. Hyperpigmentation can also occur due to inflammatory response - ‘areas of brown spots .’ It differs from dermatophytes because it is less pruritic (itchy)
111. A patient is 28 years old. He has been suffering from mental disorder since he was 22. His current condition has changed acutely: for 3 days the patient has been refusing to leave his home. He claims that there is a ”telepathy” occurring between him and other people, through which he receives ”thoughts of strangers” and transmits his own thoughts for everyone to hear. He thinks his thoughts and actions are manipulated through this ”telepathy”. Make the preliminary diagnosis:

Explanation

Schizophrenia is a mental disorder characterised by disturbances in thought and verbal behaviour, perception, motor behaviour and relationship to the external world. The different clinical types include; Paranoid, disorganised/ hebephrenic, Catatonic, residual or latent, undifferentiated schizophrenia. The described patient is said to have a paranoid schizophrenia; it is characterised by having delusions ( an unrealistic belief) of persecution, reference, control. These delusions are usually well connected and also have no disturbance  of speech and motor behaviour.

A catatonic episode is characterised by a marked disturbance of motor behaviour.

 

Organic delirium is characterised by clouding of consciousness and disorientation.

112. A 3-year-old child presents with sharp deterioration of his general condition. He has a history of purulent otitis. His temperature is now 38.5oC. The left leg is pressed to the torso, active movements are absent, the lower third of the thigh and knee joint are thickened, hyperemic, with localized fever. Axial load leads to acute discomfort of the patient. What is the most likely diagnosis?

Explanation

 

The patient has a history of purulent otitis ( a purulent inflammation of the middle ear) mostly as a result of an infection (confirmed by the increased temperature). From the description above, we can draw out that the patient has an infection around the knee joint ( knee joint is thickened, hyperemic, with localized fever). Osteomyelitis ( a rare infection of the bone) is the best possible choice.

113. A 19-year-old student was urgently hospitalized due to marked dyspnea and chest pain on the left. Her body temperature is 38.8oC. She has been presenting with these signs for 3 days. Respiratory rate is 42/min., shallow. Percussion sound is dull to the left from the center of the scapula, no respiration can be auscultated. The left heart border is displaced outwards by 3 cm. Embryocardia and heart rate of 110/min are observed. Palpation of the right subcostal area is painful. What urgent measures should be taken in this case?

Explanation

114. A 20-year-old woman, gravida 2, para 1 has been in labor for 4 hours. Her condition is satisfactory. Moderately painful contractions occur every 3 minutes and last for 35-40 seconds. The waters have not burst yet. The fetus is in longitudinal position. Fetal heartbeats are 136/min., clear and rhytmic. Major segment of the fetal head is engaged to the pelvic inlet. Vaginal examination shows smooth cervix of 6cm, amniotic sac is intact, sagittal suture is in the left oblique diameter, occipital fontanel is on the right near the symphysis pubis. What stage of the labor is it?

Explanation

The first stage of labor is considered to last from the onset of regular uterine contractions to full dilation of the cervix. divided into three phases: a latent phase, an active phase, and a transition phase. During the latent phase there is more progress in effacement (stretching and thinning) of the cervix and little increase in descent. During the active phase and the transition phase there is more rapid dilation of the cervix and increased rate of descent of the presenting part.

The second stage of labor lasts from the time the cervix is fully dilated to the birth of the fetus. It takes an average of 20 minutes for a multiparous woman and 50 minutes for a nulliparous woman.

The third stage of labor lasts from the birth of the fetus until the placenta is delivered.

 

The presented cervix is 6cm ( dilated to an extent), Major segment of the fetal head is engaged to the pelvic inlet - indicating either an active or transition phase.

115. A 1.5-month-old child on breastfeeding presents from birth with daily vomiting, irregular liquid foamy feces, and flatulence, which are resistant to antibacterial and probiotic therapy; no increase of body mass is observed. The child’s condition improved, when breastmilk was substituted. What pathology is it?

Explanation

 

The baby presented with the above stated symptoms only when  given breast milk. Meaning the symptoms are as a result of the constituents of the breast milk (lactose). This baby is termed ‘ lactose intolerant’ due to the inability to digest products containing lactose. This is due to the deficiency of the enzyme Lactase responsible for the breakdown of lactose into glucose and galactose.

Functional dyspepsia, also known as non-ulcer dyspepsia or indigestion, is a term used to describe a group of symptoms affecting the gastrointestinal tract, including stomach pain or discomfort, nausea, bloating and belching

116. A 72-year-old man complains of lower extremity edema, sensation of heaviness in the right subcostal area, dyspnea in rest. For over 25 years he has been suffering from COPD. Objectively: orthopnea, jugular venous distention, diffuse cyanosis, acrocyanosis. Barrel chest is observed, on percussion there is vesiculotympanitic (bandbox) resonance, sharply weakened vesicular respiration on both sides, moist crepitant crackles in the lower segments

Explanation

117. A burn victim with flame burns of the IIIA-B and IV degrees on his face, neck, and anterior surface of the thorax was brought into the admission room. The hairs in his nostrils are burnt, his labial and glossal mucosa are gray-white. The voice is hoarse; respirations are frequent and shallow; the patient has trumpetlike cough that produces soot-streaked sputum. The signs of respiratory failure were progressing, while the patient was being transported into the intensive care unit. What emergency care must be provided to this patient?

Explanation

The above patient is said to have burns ranging from 3A- 4 in severity; it is also noted that respiration are frequent and shallow, the signs of respiratory failure are progressing whilst the patient is in an intensive care state; this implies that to avoid a complete respiratory failure, tracheal intubation with mechanical ventilation should be performed on the patient. Intubation is done when one cannot maintain their airway on their own due to anesthesia or illness. 

 

A tracheostomy is a procedure in which a doctor surgically makes an incision in the trachea, sometimes called the “windpipe.” Tracheostomy procedures are performed when there is an obstruction in the airway and intubation is medically not possible

118. A 72-year-old man with pnaumonia complains of marked dyspnea, chest pain, severe cough with expectoration, to is 39.5- 40oC, no urination for a whole day. Objectively the patient is conscious. Respiratory rate is 36/min. Over the right lower pulmonary lobe percussion sound is dull; on auscultation there is bronchial respiration and numerous moist crackles. Blood pressure is 80/60 mm Hg. Heart rate is 120/min. Heart sounds are muffled, there is tachycardia. What tactics should the family doctor choose in the management of this patient?

Explanation

The above patient presents with pneumonia ( an infection of the lungs). He is currently experiencing a bacteremia ( spread of bacteria in the blood) characterised by;  increased temperature, heart rate, breathing rate; a low blood pressure, reduced blood supply to certain organs eg, kidney ( confirmed by the absence of urine for a day). This  patient should be immediately taken to the ICU with major steps taken to prevent a septic shock.
119. Estimation of community health level involved analysis of a report on diseases registered among the population of district under charge (reporting form 12). What index is calculated based on this report?

Explanation

120. A 72-year-old woman suffers from diabetes mellitus type II, concomitant diseases are stage II hypertension and stage IIB heart failure. She takes metformin. Hypertensic crisis had occurred the day before, after which the patient developed extreme weakness, myalgias, thirst, dry mouth, polyuria. BP is 140/95 mm Hg, heart rate is 98/min., no edemas or smell of acetone detected. What measures should be taken to prevent development of comatose state in the patient?

Explanation

121. The body of a 24-year-old woman with probable signs of poisoning has been found on the street. Forensic medical examination was requested by an investigator during examination of the site and the body. According to the Criminal Procedure Code currently in force in Ukraine, forensic medical examination is required when it is necessary to determine the:

Explanation

According to the Criminal Procedure Code currently in force in Ukraine, forensic medical examination is required to determine the cause of death.
122. It is the 3rd day after the normal term labor; the infant is rooming-in with the mother and is on breastfeeding. Objectively: the mother’s general condition is satisfactory. Temperature is 36.4oC, heart rate is 80/min., BP is 120/80 mm Hg. Mammary glands are soft and painless; lactation is moderate, unrestricted milk flow. The uterus is dense, the uterine fundus is located by 3 fingers width below the navel. Lochia are sanguino-serous, moderate in volume. Assess the dynamics of uterine involution:

Explanation

Involution is the term used to describe the physical reduction in size of the uterus and cervix after parturition. This process begins immediately after expulsion of the placenta with contraction of the uterine smooth muscle. The uterus, which at full term weighs approximately 11 times its prepregnancy weight, involutes to approximately 500 g by 1 week after birth and to 350 g by 2 weeks after birth. At 6 weeks it weighs 50-60 g. 

 

Subinvolution is the failure of the uterus to return to a nonpregnant state. The most common causes of subinvolution are retained placental fragments and infection.

123. A 27-year-old man was hospitalized in severe condition 50 minutes after receiving a penetrating wound to the left side of the chest. Objectively the patient is in a stupor, his skin is pale and acrocyanotic. Pulse is 120/min., of poor volume, weak. Blood pressure is 80/40 mm Hg. Heart sounds are muffled, cardiac borders are markedly expanded. In the III intercostal area along the parasternal line on the left there is a stab-incised wound. Plain chest X-ray shows enlarged heart shadow with smoothed out waist of the heart, there is hemothorax on the left to the 5th rib. What contributes the most to the severity of the patient’s condition?

Explanation

Recall that the patient presents with a very rapid and weak pulse (120/min), muffled heart sounds and an enlarged  heart shadow with smoothed out waist of the heart on x ray ( thus confirming an accumulation of fluid). Cardiac tamponade is the impaired pumping ability of the heart due to accumulation of fluid in the pericardium. N/B the the patient is in a state of stupor, his skin appears pale and acrocyanotic (due to the restricted flow of blood); of the various illnesses he currently has, the major cause of his current state is the Cardiac tamponade ( it limits the amount of blood being pumped).
124. A 59-year-old patient suffering from hypertension was delivered to the hospital with complaints of acute headache, nausea, recurrent vomiting. On examination she presents with acute meningeal symptom complex. BP is 185/105 mm Hg. What additional examination would you recommend to the patient in the first place?

Explanation

 

It is well noted that on examination, the patient presents with ‘Meningeal Symptom’ (stiff neck, sensitivity to light etc). To understand the cause of these meningeal signs, a lumbar puncture is carried out inorder to obtain the cerebrospinal fluid for analysis.

125. During regular examination of a 2-yearold boy, he presents with enlarged left kidney, painless on palpation. The right kidney was undetectable on palpation. Excretory urography shows no contrast on the right. Cytoscopy detected hemiatrophy of the urinary bladder trigone, the right ureteral orifice is not detected. What pathology is it?

Explanation

An agenesis is the failure of an organ to develop. From the above analysis, we observe an undetected right kidney on palpation; an absent contrast on the right during excretory urography indicates no organ located ( supported by an incomplete bladder trigone) - coupled with the patient’s age, we can conclude that development of the right kidney never took place. A compensatory hypertrophy/ hyperplasia of the left kidney occured in order to meet the body’s requirement.
126. A 5-year-old child has body temperature risen up to febrile numbers, suffers from inertness, weakness. Examination revealed hemorrhage on the skin of limbs and torso. Enlargement of cervical and axillary lymph nodes can be detected. The liver is 4 cm below the costal arch; the spleen is 6 cm below the costal arch. Blood test: erythrocytes - 2.3 · 1012/L, Hb- 60 g/L, platelets - 40 · 109/L, leukocytes - 32.8 · 109/L, eosinophiles - 1%, band neutrophiles - 1%, segmented neutrophiles - 12%, lymphocytes - 46%, monocytes - 1%, blasts - 40%, Duke’s bleeding time test result is 9 min. What examination is necessary to make the diagnosis?

Explanation

 

Notice that from the above analysis, the platelet level is relatively low (150 − 400 · 109/L is the normal). This is confirmed in the result of the duke test being 9 minutes - prolonged ( normal is 2-5 ). The  Duke test is done to check the bleeding time , it is a platelet function test. An abnormality indicates diseases such as von Willebrand, thrombocytopenia, DIC etc.  Recall that platelets originate from large bone marrow cells  and a bone marrow biopsy will be most helpful in analysis amongst all the above listed.

127. A 36-year-old man complains of marked dyspnea and cardiac pain. He ascribes his disease to the case of influenza that he had 2 weeks ago. Objectively he leans forward when sitting. The face is swollen, cyanotic, cervical veins are swollen. Heart borders are extended on the both sides, heart sounds are muffled, heart rate = Ps = 118/min., BP is 90/60 mm Hg. Blood test: ESR is 16 mm/hour. ECG shows low voltage. X-ray shows trapezoidal cardiac silhouette and signs of pulmonary congestion. Choose the treatment tactics:

Explanation

We can pick out: Extended heart borders, low b.p and an increased pulse . These three signs are known as Beck’s triad. They indicate a Cardiac Tamponade ( the impaired pumping ability of the heart due to accumulation of fluid in the pericardium). Note that on the x-ray, a trapezoid cardiac silhouette sign was present- this means that the normal cardiac borders were lost ( thus confirming one of the earlier stated signs). The most suitable approach  should be a pericardial puncture ( pericardiocentesis) to remove the fluid in the pericardium.
128. A 25-year-old patient is not married and has sexual relations with several partners. During the last 3 months he noticed small amount of mucoserous secretions produced from urethra. Subjectively: periodical itching or burning pain in urethra. Two months ago pain in knee join developed. Possibility of trauma or exposure to cold is denied by the patient. During the last week eye discomfort is noted - lacrimation and itching. What provisional diagnosis can be suggested?

Explanation

Reactive arthritis is an autoimmune condition that develops in response to an infection in another part of the body. Coming into contact with bacteria and developing an infection can trigger reactive arthritis. It has symptoms similar to various other conditions collectively known as "arthritis,". It is caused by another infection and is thus "reactive". The symptoms of reactive arthritis very often include a combination of three seemingly unlinked symptoms—an inflammatory arthritis of large joints, inflammation of the eyes (conjunctivitis and uveitis), and urethritis. A useful mnemonic is "the patient can't see, can't pee and can't bend the knee" or "the patient can't see, can't pee and can't climb a tree". Also known as Reiter’s syndrome.
129. A woman complains of weight gain, chills, edema, xeroderma, somnolence, difficulties with focusing. Objectively: height is 165 cm; weight is 90 kg; body proportions are of female type, to- 35,8oC, heart rate - 58/min., BP105/60 mm Hg. Heart sounds are weakened, bradycardia is observed. Other internal organs have no alterations. Thyroid gland cannot be palpated. Milk secretion from mammary glands is observed. Hormone test revealed increased levels of thyroid-stimulating hormone (TSH) and prolactin, and decreased level of thyroxine (4). What is the cause of obesity?

Explanation

Hypothyroidism is an endocrine disorder in which the thyroid gland does not produce enough thyroid hormones. It is classified under congenital and acquired. The acquired form is further divided into Primary, secondary and tertiary forms.

Primary Hypothyroidism: Occurs due to thyroid gland disturbances. This can be as a result of environmental factors, autoimmune causes e.g. Hashimoto, radiation etc. Characterised by low levels of T3 triiodothyronine and T4 thyroxine.

Secondary hypothyroidism: occur due to pituitary gland disorders. In this form, thyroid stimulating hormone (TSH) is low.

Tertiary Hypothyroidism: Occurs due to hypothalamic related disorders. Characterised by a low level of thyroid releasing hormone.

 

Symptoms include intolerance to cold, hair loss, fatigue, brittle nails, weight gain, bradycardia etc. sometimes referred to as Myxedema.

130. A 54-year-old patient complains of weakness, jaundice, itching skin. Disease onset was 1.5 months ago: fever up to 39oC appeared at first, with progressive jaundice developed 2 weeks later. On hospitalisation jaundice was severely progressed. Liver cannot be palpated. Gallbladder is enlarged and painless. Blood bilirubin is 190 mcmol/L (accounting mainly for direct bilirubin). Stool is acholic. What is the most likely reason for jaundice in this patient?

Explanation

Prehepatic/ hemolytic - occurs due to increased breakdown of RBC eg, hemolytic disease of newborn. There is an increased level of unconjugated bilirubin. Stool and urine colour are normal.

Hepatic/ parenchymatous  - Occurs in diseases affecting the liver parenchyma eg, cirrhosis, hepatitis etc. There is an increase in both unconjugated and conjugated bilirubin. Urine appears dark and faeces pale.

Post hepatic/ obstructive/ mechanical jaundice - pathology lies after conjugation of bilirubin and is caused by obstruction of biliary path. Conjugated bilirubin is accumulated, Urine is dark and faeces pale or acholic. Seen in disease such as cholelithiasis.
131. A 23-year-old man came to the surgeon with complaints of pain, redness of the skin, and swelling in the area of his proximal interphalangeal joint of the III finger on the right hand. Six days ago he pricked his finger with a wire. Objectively the III finger on the right hand is swollen, hyperemic, prominent in the projection of interphalangeal joint, sharply painful on touch and during movements. Finger mobility is reduced. Fluctuation sign is present. What diagnosis corresponds to the given clinical presentation?

Explanation

A Panaritium or Whitlow is an acute purulent inflammation of the tissues of the finger or toes. In inflammatory skin lesions or inflammation of injuries, the process quickly spreads along connective strands on deep tissue tendon, bone formation joints. Panaritium are classified based on depth: Superficial ( paronychia, cutaneous, subcutaneous) and deep (bone, joint, tendon, pandactylitis.)

Articular panaritium is an Inflammation of the capsule of the interphalangeal joint (notice that the patient presents with swelling of the interphalangeal joint and presence of prominent projections). A bone panaritium involves the osseous/bony tissue.

 
132. A 28-year-old woman complains of nausea, stomachache, pain in her tongue, and liquid feces. Three days ago she ate poorly salted pike caviar. Objectively her skin is pale, the tongue looks ”lacquered” (bald tongue). Pulse is 100/min., with muffled heart sounds and systolic murmur over the cardiac apex. Blood pressure is 95/50 mm Hg. The liver is enlarged by 3 cm. Hemogram shows anemia, eosinophils - 18%. Oval helminth eggs were detected in feces. Make the provisional diagnosis:

Explanation

Diphyllobothrium latum is an intestinal tapeworm, known as the human ‘broad’ tapeworm. It is the largest tapeworm found in man and the causative agent of Diphyllobothriasis. The infection caused by D. latum is due to the ingestion of raw, poorly cooked or pickled fresh water fish (caviar). The symptoms associated with D. latum infection may be absent or minimal with eosinophilia. There may be occasional intestinal obstruction, diarrhoea, and abdominal pain. The most serious symptom is the onset of pernicious anaemia. This is due to a vitamin B12 deficiency, caused by excessive absorption of the vitamin by the adult worm and the absorption of cobalamins from the host intestine.

 

The adult form of Taenia solium and T. saginata cause taeniasis, T. solium larvae cause cysticercosis. They are differentiated by the number of proglottids/ uterine branches 7-12 for T. solium and 17-35 for saginata. Ascariasis is caused by ascaris lumbricoides.

133. A 30-year-old woman complains of milk discharge from her breasts and no menstruation for the last 5 months. One physiologic childbirth was 4 years ago. There are no maldevelopments of mammary glands. Bimanual examination revealed diminished uterus and normal sized ovaries. MRIscan shows no brain pathologies. Thyroid stimulating hormone is within normal limits. Serum prolactin is high. What is the most likely diagnosis?

Explanation

The woman presents with milk discharge from the breast and a high level of serum prolactin. The function of prolactin  is to stimulate the production of breast milk and occurs normally in females after childbirth; It also disrupts normal menstruation cycle in females as seen in the described patient. Note that the results of the MRI of the brain showed no pathology thereby ruling out the options of pituitary adenoma and sheehan syndrome.
134. A 25-year-old woman during selfexamination detected a tumor in the upper external quadrant of her right mammary gland. On palpation: painless, dense, mobile growth 2 cm in diameter is detected in the mammary gland; no changes in the peripheral lymph nodes are observed. On US of the mammary glands: in the upper external quadrant of the right mammary gland there is a space-occupying lesion of increased echogenicity 21х18 mm in size. The most likely diagnosis is:

Explanation

 

An adenoma is a benign epithelial tumor from the epithelium of the glands and glandular organs e.g. breast, thyroid gland, ovaries etc. A fibroadenoma is a benign nodular proliferation and not a true neoplasm (cancer); it presents as a mobile lump in the breast of young women.

Breast cysts are fluid filled cavities common in females approaching menopause

135. A 45-year-old woman underwent one year ago mastectomy followed by chemo- and radiation therapy. She now complains of dyspnea at rest and temperature up to 37.2oC. Her general condition is severe, acrocyanosis is observed. The right side of her chest practically does not participate in respiration. Percussion reveals a dull sound below the 3rd rib; auscultation detects acute weakening of the respiratory sounds. Pleural puncture on the right has yielded a large amount of hemorrhagic exudate. What complication has developed in the patient?

Explanation

 

Realise that the woman had a mastectomy followed by chemo- and radiation therapy which implies she has a carcinogenic history. A current case of increased temperature, dyspnea even while at rest, non participation of the right side of chest during respiration, coupled with the result from percussion and auscultation indicate Pleuritis/ pleurisy of the right lung. Pleuritis is an inflammation of the lining of the lung. It can be as a result of a viral infection, lung cancer, pneumonia etc. The current case of pleuritis is most likely a complication of the previously mentioned carcinogenic issue.

136. A 37-year-old man suffers from attacks of unconsciousness, dyspnea during physical exertion, periodical sensations of heart rate disorder. Father of the patient died suddenly at the age of 45. Objectively: heart rate is 90/min., BP is 140/90 mm Hg. On heart US: ejection fraction - 55%, significant myocardium thickening of the left ventricle and interventricular septum. What drug should be prescribed for the treatment?

Explanation

 

Note that The patient above presents with no kidney related issue thereby ruling out the option of Enalapril ( an ACE inhibitor), furosemide (loop diuretic) and hydrochlorothiazide (thiazide diuretic) - these drugs will be the first to be considered in a case of hypertension coupled with a kidney related  issue. Bisoprolol ( beta blocker) is the best possible choice, It is a beta 1 selective blocker and acts by competitively blocking adrenaline’s stimulation of beta 1 adrenergic receptors causing a reduction of heart rate and increase in contractility.

137. A 60-year-old man complains of discomfort when swallowing solid food, which he has been observing for a month. He changed his diet to semiliquid food products. At first the discomfort had abated but later it renewed despite the change in the diet. The patient developed gaseous eructation and hoarse voice. What examination should be performed to clarify the diagnosis?

Explanation

The above patient firstly presented with a partial obstruction which later progressed to a complete obstruction. We can tell this by the inability to consume liquid food after a period of time. To confirm the cause and area of obstruction, an fibroesophagoduedenoscopy should be carried out on the patient; this involves a camera being attached to a tube and passed along the GIT so as to get a better view of its internal structures.

 

A urea breath test is carried out on a patient suspected with gastric/duodenal ulcers caused by H.pylori. Most patients will present with an epigastric pain.

138. A 22-day-old infant developed subcutaneous red nodes from 1.0 to 1.5 cm in size on the scalp; later the nodes suppurated. Temperature increased up to 37.7oC, intoxication symptoms appeared, regional lymph nodes enlarged. Complete blood count: anemia, leukocytosis, neutrocytosis, increased ESR. What diagnosis will you make?

Explanation

Pseudofurunculosis, is observed in neonates and infants. The disease begins with the appearance of superficial pustules in the ostium of sweat glands (periporitis). Caused by staphylococcus aureus, which penetrates in the depth of the sweat gland and causes the forming of deep indurated painful nodules. The nodules are very similar to furuncles, but they have not got core in the center (hence the name pseudofurunculosis).

 

Vesiculopustulosis is a widespread purulent disease, which appears in the first years of life. In the ostium of the sweat glands numerous pustules appear, filled with white yellow matter, the size of a pin head to a small pea, they do not merge with each other and are surrounded by bright edematous circles.

139. A 32-year-old woman complains of general fatigue, low-grade fever persisting for 4 months, lumbar pain, and dysuria. Anamnesis includes frequent acute respiratory diseases, overexposure to cold, lowcalorie diet, a case of pulmonary tuberculosis in childhood. Clinical urine analysis: pH4.8, leukocyturia, hematuria. Complete blood count: leukocytosis, lymphocytosis, increased ESR. Urography concludes: dilatation of renal pelvis and calyceal system of both kidneys, foci of calcification in the projection of right kidney parenchyma. What is the most likely diagnosis?

Explanation

140. A 45-year-old woman came to the maternity clinic with complaints of periodical pains in her mammary glands that start 1 day before menstruation and stop after the menstruation begins. Palpation of the mammary glands detects diffuse nodes predominantly in the upper outer quadrants. What is the most likely diagnosis?

Explanation

A fibrocystic mastopathy is a benign condition of the breast, it is the most common disorder of the female breast. Fibrocystic change is characterised  by hyperplastic overgrowth of components of the mammary unit, i.e lobules, ductules and stroma. In this condition, there are 4  characteristic features namely: Fibrosis (an increase in the amount of collagen rather than true growth of fibrous tissue), Adenosis ( an increase in the number of lobules and in the size of existing lobules), Cyst formation ( cyst are lined by flattened epithelium derived from lobal ductal unit and are filled with watery fluid) and Fibrocystic changes ( most commonly epithelial hyperplasia).

 

A mastitis is simply an inflammation of the mammary gland.

141. A woman complains of temperature increase up to 39oC, sharp pains in her lower abdomen, and sanguinopurulent discharge from her genital tracts. From her case history it is known that 6 days ago she underwent illegal abortion. Objectively her blood pressure is 100/60 mm Hg, pulse is 110/min. Abdominal rigidity, rebound tenderness (Bloomberg’s sign), and painful palpation of the lower abdomen are observed. On bimanual examination the uterus is enlarged up to 7 weeks of pregnancy, painful, and soft; posterior vaginal fornix overhangs. Make the diagnosis:

Explanation

Pelviperitonitis is the inflammation of the peritoneum ( serous membrane) surrounding the fallopian tubes and uterus. Observe that the patient presents with signs of peritonitis which includes; painful palpation of the abdomen, rebound rigidity ( a positive shotkin- blumberg sign) and abdominal rigidity. The disease is mostly as a result of a bacterial infection of the female genital tract that spread lymphogenously or hematogenously. The Disease begins sharply, is characterized by high temperature of a body (38 — 39 ° above), deterioration in the general state, severe pains in the bottom of the stomach, increase of pulse, abdominal distention etc.

 

An adnexitis is the inflammation of the uterus and its appendages ( ovaries and fallopian tubes).

142. An 8-year-old girl with complaints of painful urination, frequent low-volume urination, and leukocyturia was diagnosed with acute cystitis. 10 days before the disease onset she was treated by the gynecologist for acute vulvitis. 5 days ago she presented with mild catarrhal symptoms. Her mother ascribes the child’s disease to her overexposure to cold. Specify the most likely infection route:

Explanation

Notice that the patient presents with a history of vulvitis ( inflammation of the vulva) and a urinary tract infection. The more common mode of UTIs is through the ascending pathway, where fecal flora gain access to the urinary tract via colonization of the urethra. 

 

Descending infections are the result of hematogenous spread of bacteria from a primary source located elsewhere in the body.

143. A 74-year-old patient visited a urologist with complaints of pain above the pubis and inability to urinate for 8 hours. At home he had taken antispasmodics and had a warm bath but no improvement occurred. Objectively: abdomen is soft and painful above the pubis; dullness of percussion sound is observed above the pubis. Murphy’s (Pasternatski’s) punch sign is negative on the both sides. What condition does the patient have?

Explanation

 

A negative murphy’s punch (pasternetsky’s sign) rules out an infection around the kidneys. An inability to voluntarily pass out urine  especially when the bladder is full is termed an Acute urinary retention. Chronic form is usually as a result of inadequate bladder emptying over a long period of time . Notice that the patient's inability has only lasted about 8 hours thus an acute case.

144. An excavator operator with 20 years of work experience at the opencast ore mine undergoes regular medical examination. He presents with signs of pneumoconiosis. What type of pneumoconiosis is the most likely in this case?

Explanation

 

Pneumoconiosis is a restrictive lung disease  caused by the inhalation of dust leading to fibrosis. Depending on the type of dust inhaled, different types exist. They include; Siderosis (iron ore), byssinosis (cotton), anthracosis (coal), Asbestosis (asbestos), silicosis (silica dust) etc.

145. A 55-year-old woman came to a gynecologist with complaints of leukorrhea and bloody discharge from the vagina after 5 years of menopause. Anamnesis states no pregnancies. Bimanual examination: the uterus and uterine appendages are without changes. During diagnostic curettage of the uterine cavity the physician scraped off enchephaloid matter. What is the most likely diagnosis in this case?

Explanation

  1. Basically, there are three forms of cancer of the uterus (endometrium): 

      1. Fibrous carcinoma or scirrhous, hard cancer

      2. Medullary or encephaloid, or soft cancer 

      3. epithelioma, or chancroid.

 

Observe that on curettage of the uterine cavity, an encephaloid matter was found- this indicates the presence of an endometrial (uterine) carcinoma.

146. A 5-year-old boy complains of severe dyspnea and sensation of lack of air. Objectively the child assumes orthopneic position, presents with marked peripheral cyanosis, drumstick fingers, nail plates resembling a ”clock face”, the borders of cardiac dullness are bilaterally extended; coarse systolic murmur can be detected over the whole surface of the heart and is especially pronounced in the II intercostal area on the left near the sternum. What disease can be characterized by such presentations?

Explanation

147. A 10-year-old boy is delivered into a polytrauma unit after he received a blunt trauma of the thorax, having fallen from the bicycle. Upon hospitalization his blood pressure is 110/80 mm Hg, heart rate is 96/min. Chest X-ray is noncontributive to the diagnosis. Echocardiogram shows free liquid in the pericardial cavity, in the amount of up to 100 ml. In an hour after the hospitalization the patient started to develop increasing signs of heart failure: jugular venous distention, decreased blood pressure down to 90/70 mm Hg, tachycardia up to 120/min. On auscultation muffled heart sounds. What would be the primary tactics of a physician?

Explanation

Notice that the result from the echocardiogram shows the accumulation of fluid in the pericardium thereby indicating a pericardial effusion. As a result of this, the intrapericardial pressure will be increased leading to an adverse effect on the heart. In order to remove the accumulated fluid, a Pericardiocentesis (pericardial puncture) is carried out. This is a medical procedure in which a needle is used in aspirating (removing) the excess pericardial fluid.
148. A patient has gradually lost consciousness. The skin is pale and dry. There is smell of ammonia from the mouth. Respirations are deep and noisy. Heart sounds are muffled, pericardial friction rub is present. Blood pressure is 180/130 mm Hg. Blood test: Нb- 80 g/L, leukocytes - 12 · 109/L, blood glucose - 6.4 mmol/L, urea - 50 mmol/L, creatinine - 1200 mcmol/L, blood osmolality - 350 mOsmol/kg H2O. No urinary excretion. Make the diagnosis:

Explanation

Observe that there is the presence of an ammonia smell and from the biochemical analysis, urea level is - 50 mmol/L ( 3.33 - 8.32 mmol/L normal). Recall that ammonia is converted to urea (via the ornithine cycle) which is later excreted through the urine. This patient has no urinary excretion leading to the accumulation of urea. Urea is a very toxic substance, it possesses the potential of crossing the blood brain barrier, causing a toxic effect on the brain leading to a uremic coma.

 

Hyperglycemic coma will be experienced in very high levels of blood glucose while acute renal failure can be seen when the kidneys  aren't able to meet up the excretory requirements of the body.

149. A 26-year-old woman has been undergoing treatment for community-acquired pneumonia for 10 days. It is known that her husband had been treated for drug addiction. Sequential intravenous administration of Amoksiklav (Amoxicillin+Clavunate) + Levofloxacin combination and vancomycin in the prescribed dosage was ineffective. Within the last two days the patient’s dyspnea and intoxication acutely exacerbated, bilateral pulmonary infiltrates are observed. What is the most likely cause of the medication ineffectiveness?

Explanation

The husband being a drug addict (most likely an intravenous drug user) raises strong suspicion for infection with HIV.

HIV is an immunosuppressive disease and patients suffering from  this disease are prone  to having opportunistic  infections like pneumocystic pneumonia (PCP) caused by pneumocystis jirovecii. 

 

PCP is a yeast like fungus and doesn't respond well to antibacterial medications like amoxiclav. It is best treated with Trimethoprim-Sulfamethoxazole.

150. A 35-year-old man complains of persisting enlargement of his peripheral lymph nodes that cause him no discomfort. The case history states that the first lymph nodes to enlarge were cervical, supraclavicular, and axillary; new groups of lymph nodes emerged. Objectively the lymph nodes are soft and elastic on palpation, enlarged, painless, not fixed to the surrounding tissue. What examination method would be the most informative for early diagnostics of this disease?

Explanation

A needle biopsy is a procedure that uses a thin hollow needle and a syringe to extract cells, tissues or fluid from suspicious parts of the body; the material is studied under the microscope to determine the cause of the abnormality.

 

A needle biopsy may be called for when a doctor feels an abnormal lump or when imaging scans show the presence of abnormalities in an area of the body. Lumps of the breast, thyroid or lymph nodes are often checked using a needle biopsy.

151. A 32-year-old woman complains of body weight loss despite her increased appetite, nervousness, and tremor of the extremities. Objectively: the skin is moist; the thyroid gland is diffusely enlarged, painless, soft, and mobile. Blood test: increased level of T3, T4, and decreased thyroid-stimulating hormone (TSH). What is the most likely diagnosis?

Explanation

Diffuse toxic goiter or grave’s disease is an autoimmune disease characterised by an increased amount of thyroid hormones ( T3- triiodothyronine and T4- thyroxine). Symptoms include; loss of weight, increased sweating, tachycardia, tremors of extremities, increased appetite etc.

 

Hashimoto’s thyroiditis is a destructive autoimmune thyroiditis leading to hypothyroidism.

152. A 64-year-old man suddenly sensed pain in his occipital area, dizziness, general weakness. He has a 15-year-long history of hypertension. Objectively the skin and mucosa are of normal color. Auscultation reveals vesicular respiration across the lung surface. At the cardiac apex the I heart sound is weakened, the II heart sound is accentuated over the aorta. Pulse is 84/min., blood pressure is 180/100 mm Hg. Other body organs and systems are unaffected. What drug should be prescribed in the first place?

Explanation

 

Angiotensin Converting enzyme Inhibitor drugs ( ACE inhibitors) are a group of drugs used in cases of hypertension and heart failure. The drug acts on the Renin-angiotensin-aldosterone system by inhibiting the enzyme responsible for the conversion of Angiotensin I to Angiotensin II. N/B Angiotensin II is a potent vasoconstrictor; during this process, bradykinin a potent vasodilator is released. The examiner outlined 3 ACE inhibitors (captopril, ramipril and perindopril). Of the three listed ramipril and perindopril are prodrugs  ( administered as inactive drugs but are converted into active form during metabolism). Captopril is not administered as a prodrug and can be taken orally ( the other ACE inhibitor of this calibre is Lisinopril).

153. During winter epidemics of influenza caused predominantly by virus А/California/04/2009 (H1N1), on the 2nd day after the disease onset a 30-year-old hospitalized man presented with high fever, dry cough, myalgia, headache, and general weakness. What should be prescribed as etiotropic treatment in this case?

Explanation

An etiotropic treatment is one aimed at the causative agent. In this case a Virus - Influenza A. Oseltamivir is an antiviral drug aimed at the treatment of influenza A and B. It acts by inhibiting influenza neuraminidase ( an enzyme needed for influenza replication). Zanamavir also has this function.

 

Acyclovir is also an antiviral drug used mainly in treatment of the herpes Simplex virus and Varicella zoster virus.

154. A 24-year-old pregnant woman on her 37th week of pregnancy has been delivered to a maternity obstetric service with complaints of weak fetal movements. Fetal heartbeats are 95/min. On vaginal examination the uterine cervix is tilted backwards, 2 cm long, external orifice allows inserting a fingertip. Biophysical profile of the fetus equals 4 points. What tactics of pregnancy management should be chosen?

Explanation

A biophysical profile is a prenatal test used to check the well being of a baby. It involves using an ultrasound in evaluating the fetal heart rate, breathing, movement ,muscle tone and amniotic fluid level combined with a non-stress test for checking the fetal heart rate. Points are allocated for every measurement taken. This test is recommended for pregnant women at high risk of complications and a tendency of pregnancy loss. In most cases, a low biophysical profile score might indicate an early or immediate child delivery. 

 

Note that the score of the above profile is 4 (indicating an immediate delivery).

155. During regular preventive gynecological examination a 30-year-old woman was detected to have dark blue punctulated ”perforations” on the vaginal portion of the uterine cervix. The doctor suspects endometriosis of the vaginal portion of the uterine cervix. What investigation method would be most informative for diagnosis confirmation?

Explanation

A colposcopy is a simple procedure used to look at the cervix, the lower part of the womb at the top of the vagina ( Colposcopy is like a microscope put near the vagina to allow more detailed examination of the cervix). It's often done if cervical screening finds abnormal cells in your cervix. Under the guidance of colposcopy, a piece of the lesion will be removed (called a biopsy) and sent to the laboratory for detailed examination.

 

A hysteroscopy is used in detecting pathologies involving the Uterus. Curettage is a procedure used in removing tissues from the uterine cavity.

156. A 26-year-old woman came to a gynecologist for a regular check-up. She has no complaints. Per vaginum: the uterus lies in anteflexion, not enlarged, dense, mobile, painless. On the left from the uterus in the area of uterine appendages there is a mobile painless outgrowth that can be moved independently from the uterus. On the right the appendages cannot be detected. What additional investigation would be informative for diagnosis clarification?

Explanation

An ultrasound of the lesser pelvis is used to examine the organs of the pelvis; in females these include the uterus, cervix, vagina, fallopian tubes and ovaries. The above patient has issues with the uterine appendages and this diagnostic method will be the most informative among all listed. 

 

A colposcopy is a simple procedure used to look at the cervix, the lower part of the womb and the top of the vagina.

157. A 38-year-old man underwent surgical treatment of a wound with a suppuration focus. On the 8th day after the procedure the wound cleared of purulo-necrotic discharge and granulations appeared. However, against the background of antibacterial therapy, the patient’s body temperature persists as high as 38.5-39.5oC; chills, excessive sweating, and euphoria are observed in the patient; heart rate is 120/min. What complication of the local suppurative inflammatory process can be suspected?

Explanation

 

Sepsis is simply an inflammatory immune response triggered by infection. The above patient most likely got an infection during the surgical process; Fever, chills, excessive sweating, tachycardia are all signs of the body’s reaction to an infection. N/B sepsis can be of bacterial, fungal, protozoal or viral cause and the patient’s non-response to antibacterial therapy may indicate a non bacterial origin.

158. A man complains of high fever, pain in the area of his right mastoid bone, and purulent discharge from the right ear. One week ago he had a case of URTI. Objectively the right auricle protrudes, the skin behind the ear is hyperemic and pastose; on palpation of the mastoid bone the pain intensifies; the auditory meatus is filled with thick pus, posterosuperior meatal wall sags; the tympanic membrane is red and perforated. Make the diagnosis:

Explanation

159. A 52 year old man came to see his family physician complaining of pain in the chest. After taking history and performing physical exam the doctor decided to direct the patient to cardiologist for a consultation. What level of medical care is being proposed to the patient?

Explanation

According to levels of specialization, types of medical care include: Emergency, Primary, Secondary, Tertiary, palliative, medical rehabilitation and dentistry.

Primary medical care: involves care given by nurses, midwives, general physicians, family doctors, in polyclinics etc

Secondary medical care: When your primary care provider refers you to a specialist, you are then in secondary care. Secondary care simply means you will be taken care of by someone who has more specific expertise eg cardiologist, endocrinologist etc

Tertiary Medical Care: Once a patient is hospitalized and needs a higher level of specialty care within the hospital, he may be referred to tertiary care. Tertiary care requires highly specialized equipment and expertise.

 

At this level, you will find procedures such as coronary artery bypass surgery, renal or hemodialysis, and some plastic surgeries or neurosurgeries.

160. A 30-year-old woman made an appointment with the family doctor for scheduled vaccination of her 2-year-old child. What type of healthcare provides such medical services?

Explanation

According to levels of specialization, types of medical care include: Emergency, Primary, Secondary, Tertiary, palliative, medical rehabilitation and dentistry.

Primary medical care: involves care given by nurses, midwives, general physicians, family doctors, in polyclinics etc

Secondary medical care: When your primary care provider refers you to a specialist, you are then in secondary care. Secondary care simply means you will be taken care of by someone who has more specific expertise eg cardiologist, endocrinologist etc

Tertiary Medical Care: Once a patient is hospitalized and needs a higher level of specialty care within the hospital, he may be referred to tertiary care. Tertiary care requires highly specialized equipment and expertise.

 

At this level, you will find procedures such as coronary artery bypass surgery, renal or hemodialysis, and some plastic surgeries or neurosurgeries.

161. A 28-year-old man after car accident received a wound to the right side of his chest inflicted by a sharp metal object. A foamy liquid flows out from the wound, there are tympanitis and acutely weakened respirations in the right. Blood pressure is 70/30 mm Hg, pulse is 120/min., Hb is 28 g/L. X-ray shows collapsed right lung, horizontal fluid level is at the 3rd rib. What treatment tactics should be chosen?

Explanation

 

The above patient presents with a major trauma to the right side of the chest which is confirmed by a collapsed right lung on x ray. Thoracotomy is a surgical procedure carried out in order to have access to thoracic organs such as the lungs, heart or esophagus. This procedure is the first step in gaining access to the thoracic cavity and thus is indicated in cases of a lobectomy, major trauma of the chest, pneumonectomy. In the case of this patient, an urgent thoracotomy should be carried out in order to take out the sharp metal object.

162. A 38-year-old woman has been working as a milker for 15 years. She made an appointment with the doctor due to development of red rashes on her hands, predominantly in the interdigital space. The rashes are weeping, itching, and expanding on her skin. Examination of her hands shows her nail plates to be yellow and brittle. These presentations aggravate during work. Make the provisional diagnosis:

Explanation

Eczema (dermatitis ) is a chronic inflammatory disease of the skin characterized by the presence of red, itchy, dry scaly rashes. Occupational eczema (dermatitis) is gotten from contact with certain chemicals in the course of working. The patient in question is a milker ( a person who extracts milk from cows) and most likely gets dermatitis as a result of contact with these animals or certain chemicals.

 

N/B Scabies is caused by a mite Sarcoptes scabiei; pemphigus is an autoimmune disease (antibodies against desmoglein), sores and blisters are seen in the mouth and genitals.

163. An 18-year-old patient complains of skin rash. The patient has been suffering from this condition for 5 years. The first instance of this disease occurred after a car accident. Objectively: the patient presents with papular rash covered in silvery scales, ”thimble” symptom (small pits on the nails), affected joints. What is the most likely diagnosis?

Explanation

Psoriasis is a common skin disorder characterised by the presence of papules and plaques with silvery scaling especially on the knees and elbows. Other characteristics include acanthosis, parakeratosis and pinpoint bleeding ( Auspitz sign). The patient presents with a thimble symptom which translates into a pitting or a psoriatic nail.

 

A Panaritium or Whitlow is an acute purulent inflammation of the tissues of the finger or toes. Onychomycosis is a fungal infection of the nails; signs are white or yellow nail discoloration, thickening of the nail, and separation of the nail from the nail bed; it is also known as  tinea unguium.

164. A postparturient woman, who has been breastfeeding for 3 weeks, made an appointment with the doctor. For the last 6 days she has been feeling unwell, complains of body temperature of 38-39oC, general weakness; within the last 2 days she developed pain and redness in the area of her right mammary gland. Examination revealed her mammary gland to be significantly enlarged and deformed; breast tissue fluctuations and lymphadenitis are observed. What type of mastitis is the most likely?

Explanation

165. When her car collided with a tree, a 37- year-old woman felt sharp pain in her left hip joint. She was unable to get out of the car. Her position is forced, the hip is pressed to the abdomen, fixed, and rotated inwards; the limb is flexed in the knee, any attempt to change the position results in sharp pain. Make the diagnosis:

Explanation

As a result of the accident, the woman experienced a dislocation of the hip joint (change in positioning between the head of the femur and acetabulum); this is confirmed by the hip being pressed to the abdomen, fixed, and rotated inwards. A dislocation not complicated by an external wound or injury is a Closed Dislocation while one in which the overlying skin is injured is known as an Open dislocation.

 

Contusion of the hip is simply a bruise. Hemathrosis refers to bleeding into the joint cavity.

166. On the 9th day after childbirth the obstetric patient developed high fever up to 38oC. She complains of pain in the right mammary gland. The examination revealed the following: a sharply painful infiltrate can be palpated in the right mammary gland, the skin over the infiltrate is red, subareolar area and nipple are swollen and painful. What is your diagnosis?

Explanation

 

A breast abscess is a localised collection of pus in the breast tissue. It is usually caused by a bacterial infection. It is characterised by the presence of a lump on palpation (painful infiltrate can be palpated), swollen and painful nipple. In mastopathy, we observe changes such as swellings, nodules, cysts etc, it is hormone dependent. In cancer, there will be increased proliferation of atypical tissues.

167. During reanimation procedures an attempt to intubate trachea was made. Neither glottis nor epiglottis can be visualized via laryngoscopy. What tactics should be chosen in this case?

Explanation

168. A 48-year-old woman developed insomnia, depressive mood, anxiety, fears and suicidal thoughts after the death of her husband that occurred one month ago. During her stay in the hospital she speaks in a low voice, is depressed, anxious, avoids sleeping, refuses to eat. What medications should be prescribed in this case?

Explanation

 

The question specifies that the described lady is Depressed therefore, antidepressant is the best choice among the listed groups. Antipsychotic drugs/ neuroleptics are used in cases of Schizophrenia or bipolar disorders. Nootropics are memory  enhancing drugs.

169. A 39-year-old man suffers from chronic adrenal insufficiency and receives replacement glucocorticoid therapy (hydrocortisone - 15 mg/day). He is to undergo elective surgery for calculous cholecystitis. What medication adjustment should be made on the day of the surgery to prevent the development of acute adrenal insufficiency?

Explanation

Acute adrenal Insufficiency is a sudden failure in adrenal gland hormone production (aldosterone and cortisol). It can be triggered by stress, trauma, infection, Surgery etc. The above patient suffers from a chronic case of adrenal insufficiency and is already on hydrocortisone (a glucocorticoid); to avoid an adrenal gland crisis triggered by the surgical process, the patient should be administered an increased dose of glucocorticoid. An antibiotic regimen should be added to prevent sepsis, while large intravenous fluid should be used in case of dehydration.
170. A 22-year-old man at 18:00 developed persisting dull pain in the epigastrium. Three hours later nausea appeared, he vomited once. By the morning the pain shifted to the right iliac area. Body temperature rose to 38.6oC, developed tachycardia of 110/min. On examination there are muscle rigidity and Bloomberg’s sign (rebound tenderness) in the right iliac area of the anterior abdominal wall. Plain x-ray of the abdomen shows no fluid levels, free air under the diapragm on the right. Make the diagnosis:

Explanation

 

Anatomically, Presence of a persisting dull pain in the epigastrium indicates a gastric related disorder. On x-ray, the presence of free air under the diaphragm (pneumoperitoneum) is classically seen in cases of perforated ulcers ( one of the complications of ulcers). It occurs as a result of the escape of air from the stomach via the perforations. In cholecystitis, the pain will appear at the right upper quadratic region while a case of pancreatitis will present with pain radiating to the back.

171. 40-50 minutes after the completion of repair works conducted in a closed garage, with car engine running, the repair workers developed severe headache in the temporal area, nausea, tinnitus, vertigo, etc. These symptoms are characteristic of acute poisoning with:

Explanation

 

Note that the man works in a closed garage and carbon monoxide is the type of gas released from the engines of these vehicles. Accumulation of these gases is poisonous for health and leads to the expressed symptoms.

172. After a long drive with the window open a man developed facial asymmetry; he cannot close his right eye, his right nasolabial fold is smoothed out, movements of expression are absent on the right, there is a disturbance of gustatory sensation in the tongue on the right. No other neurological pathologies were detected. What disease can be provisionally diagnosed in this patient?

Explanation

Bell's palsy is a form of facial paralysis resulting from a dysfunction of the cranial nerve VII (the facial nerve). Signs include an impaired facial expression on the affected side, drooping of the eyelid, a change in taste, pain around the ear, and increased sensitivity to sound etc. In trigeminal neuropathy, sensation of the face is disturbed
173. On ultrasound of the thyroid gland, a 47- year-old woman presents with a hypoechoic node 1.6 cm in diameter with blurred margins and intranodular hypervascularization. The doctor suspects thyroid carcinoma. What method should be used to verify the diagnosis?

Explanation

A needle biopsy is a procedure that uses a thin hollow needle and a syringe to extract cells, tissues or fluid from suspicious parts of the body; the material is studied under the microscope to determine the cause of the abnormality.

 

A needle biopsy may be called for when a doctor feels an abnormal lump or when imaging scans show the presence of abnormalities in an area of the body. Lumps of the breast, thyroid or lymph nodes are often checked using a needle biopsy.

174. A 19-year-old young man complains of cough with expectoration of purulent sputum in amount of 100 ml per day, hemoptysis, dyspnea, increased body temperature up to 37.8oC, general weakness, weight loss. The patient’s condition has been persisting for 4 years. Exacerbations occur 2-3 times per year. The patient presents with malnutrition, pale skin, cyanosis of the lips, drumstick (clubbed) fingers. Tympanic percussion sound in the lungs, weakened respiration, various numerous moist crackles in the lower pulmonary segments on the left can be observed. In blood: erythrocytes - 3.2 · 1012/L, leukocytes - 8.4 · 109/L, ESR- 56 mm/hour. On X-ray: lung fields are emphysematous, the left pulmonary root is deformed and dilated. What is the most likely diagnosis?

Explanation

Bronchiectasis is a chronic necrotizing infection of the bronchi and bronchioles leading to or associated with abnormal dilation of these airways. Symptoms include  cough and daily  mucopurulent sputum production, often lasting for months; blood-streaked sputum (hemoptysis), dyspnea, low grade fever, pleuritic chest pain, wheezing, fever, weakness, fatigue and weight loss. Presence of moist crackles on auscultation indicates an infectious or inflammatory process; digital clubbing is seen in 2-3% of patients. A dilated and deformed left pulmonary root confirms bronchiectasis of the left lung.

 

Lung abscesses are characterized by large cavities filled with suppurative debris.

175. An 8-year-old child presents with blood pressure up to 180/100 mm Hg in the upper limbs accompanied by headaches, tinnitus, occasional nosebleeds, and high fatigability. On examination there is no pulse over the leg arteries. ECG shows left ventricular hypertrophy. MRI-scan shows aortic narrowing to 5 mm in the typical place. Coarctation of aorta is diagnosed. What kind of treatment should be prescribed in this case?

Explanation

 

The patient is diagnosed with coarctation of the aorta which refers to the narrowing of the aorta leading to reduced  blood flow to various parts of the body. Treatment approaches usually consist of surgery or a procedure called balloon angioplasty or stent placement. The various types of surgery include; Bypass graft repair, Subclavian flap aortoplasty. Resection with end-to-end anastomosis etc. Medication/ conservative treatment isn't used to repair coarctation of the aorta, but it might be used to control blood pressure before and after stent or surgery.

176. A 6-year-old girl arrived to the in-patient unit with complaints of enlargement of the lower third of her right thigh. According to the case history, she has been stepping carefully on her right leg and limping for 6 months. Blood test detected anemia. X-ray of the right thigh shows a round bone defect with clear margins resembling melting sugar in the distal metaphysis. What provisional diagnosis can be made in this case?

Explanation

 

Osteosarcomas are primary malignant tumors of bone that are characterized by the production of osteoid or immature bone by the malignant cells. Characteristics of osteogenic sarcoma include; the presence of osteoid-producing cells ( malignant osteoblasts), present as painful enlarging mass . A major sign seen on x-ray is the Codman triangle ( a subperiosteal lesion), in some literature, it is described as ‘resembling melting sugar’ or ‘sunburst pattern’.

177. A 37-year-old man working as a typesetter in a print shop complains of rapid fatigability, paroxysmal attacks of stomachache, weak drooping hands. Examination of neurological status revealed hypotrophy of the forearm muscles. Carporadial reflexes are sharply weakened. Sensitivity is not disturbed. Gums present with dark blue border. What neurological pathology is it?

Explanation

The patient presents with weak dropping hands and a weakened carporadial reflex; this sign is often seen in individuals poisoned by lead ( wrist and foot drop). Other major signs are ’ Lead lines’ on gingiva/gums ( Burton's line),abdominal colic, encephalopathy and sideroblastic anemia.

Guillain-Barre syndrome is an autoimmune condition that destroys schwann cells via inflammation and demyelination of nerve fibres. It occurs as a result of molecular mimicry and triggered by infections, stress etc. major signs include Bilateral facial paralysis and respiratory failure.

 

Shingles is gotten from varicella zoster virus infection.

178. The mother of an epileptic son complains of her son periodically presenting with irritable depression, when he becomes excited and prone to physical agression and violence. These moods last for 5-10 minutes, after which the patient exhausts himself and falls asleep. Upon waking he is depressed, sad, cannot recall his actions or recalls them only partially. What psychopathologic condition is it?

Explanation

179. A 44-year-old woman has undergone subtotal thyroid resection due to diffuse toxic goiter. On the second day after the surgery the patient’s condition deteriorated; she developed palpitations, dyspnea, sweating, and diarrhea, and became fearful The patient is anxious, her skin is moist and hot to the touch. Her temperature is 39.2oC. Heart sounds are muffled, tachycardia is observed, pulse is 160/min., blood pressure is 85/40 mm Hg. The stomach is soft and painless. What should be measured to clarify the patient’s condition?

Explanation

 

The patient presents with anxiety, tachycardia, profuse sweating,, sweating, palpitation etc the above symptoms can be seen in cases of increased secretion of hormones such as adrenaline, noradrenaline or thyroid hormones; since the patient  just underwent a subtotal thyroidectomy ( removal of part of the thyroid gland), the thyroid hormones level should be checked.

180. The dermatologist has an appointment with a 30-year-old man that complains of severely itching rashes that especially disturb him at night. The rashes developed 2 weeks ago, after he had returned from a travel. Objectively on the lateral surfaces of his fingers, hands, wrists, elbows, lower abdomen, genitals, and thighs there are paired papulovesicles, single pustules, and scratch marks. What disease can be suspected?

Explanation

Scabies is an itchy skin condition caused by a tiny burrowing mite called Sarcoptes scabiei. Intense itching occurs in the area where the mite burrows. The urge to scratch may be especially strong at night. The first time a person is exposed to the scabies mite, it can take upwards of 2 to 6 weeks for symptoms to develop. So, suspected contacts i.e. pupils of the class where the patient learns can be checked in 6 - 8 weeks (approximately 2 months) to see if they’ve manifested any symptom to the infection.

Scabies is contagious and can spread quickly through close physical contact in a family, child care group, school class, nursing home or prison. Because scabies is so contagious, doctors often recommend treatment for entire families or contact groups.

The most common site of infestation in adults and older children include: in between the fingers; around fingernails; armpits; waistline; inner parts of the wrists; inner elbow; soles of the feet; the breasts, particularly the areas around the nipples; male genitalia; buttocks; knees; shoulder blades.

Infants and young children experience infestation in other areas of the body, including the: scalp; face; neck; palms of the hands; soles of the feet.

181. A 28-year-old man, a teacher, after emotional stress developed painful muscle spasms in his right hand that occur during writing; now he has to hold the pen between the second and third fingers. He has no problems with typing or writing on the blackboard; no other motor disturbances or neurological pathologies are detected. What is the most likely diagnosis?

Explanation

182. A 32-year-old pregnant woman at the term of 5-6 weeks was vaccinated against influenza along with her whole family. At that time she was not aware of her pregnancy. The pregnancy is wanted. The woman needs an advice from the family doctor regarding the maintenance of her pregnancy, namely whether there is a risk of fetal malformations because of received vaccination. What advice should the doctor give in this case?

Explanation

 

An inactivated influenza vaccine is reliably used as a form of prophylaxis from the influenza infection; optimal terms for vaccination are october-november. Influenza vaccination is indicated in children with: chronic bronchopulmonary diseases, cardiovascular diseases, hemolytic anemias, diabetes mellitus, chronic kidney and liver diseases, HIV infection etc. Note that the flu vaccine is safe during pregnancy. The inactivated Influenza vaccine can be given to pregnant women during any trimester.

183. A 15-year-old girl suffers from systemic lupus erythematosus and has been receiving prednisolone therapy in the daily dosage of 2 mg/kg for the last 6 weeks. The plans are made to gradually lower the dosage of the medicine. No clinical signs of her disease are observed. Previously she has received no immunization against measles. Due to measles outbreak it is necessary to develop the immunity against this infection in the patient. When can she be vaccinated?

Explanation

 

For specific prevention of Measles, the MMR vaccine is used (measles, mumps and rubella). It is a live/attenuated vaccine administered at the age of 12 months, Revaccination at 4 to 6 years or at 10 to 11 years. After contact with measles, unvaccinated persons under 30 years of age without a history of measles or any contraindication for vaccination should be vaccinated  with measles vaccine not later than 72 hours after the contact.  Recall that the patient has no previous record of measles vaccination and due to the outbreak of measles, she should immediately be vaccinated ( or within the next 72 hours)

184. A 45-year-old veterinary worker has made an appointment with the doctor for regular examination. In his duties he frequently deals with animals, however he denies working with rabies-affected animals. Previously he has received no antirabic vaccination. What should the doctor recommend in this case?

Explanation

In rabies endemic areas, those at high risk of exposure to rabid animals should be given pre-exposure vaccination. These include veterinarians, health care personnel, laboratory workers, and dog catchers. The anti-rabies vaccine is an inactivated vaccine.
185. A healthy child 1 year and 5 months of age is being vaccinated against hepatitis B. The child did not receive the first dose of the vaccine previously, while in the maternity hospital. The doctor makes an individual vaccination schedule for this child and plans the administration of the next dose of the vaccine. What is the minimum interval between doses of vaccine in this case?

Explanation

A minimum interval is the shortest time between two doses in a vaccination series. The Hepatitis B vaccination is given in a three dose series; First dose at birth, second Between 1-2 months and the third dose is given between the 6th-15th month. Examples of the hepatitis B vaccines are Engerix-B and Recombivax HB; the minimum interval between the first and second doses is 4 weeks.
186. A 46-year-old man came to the surgeon’s office. He complains of twitching sensation in the wound on his left foot, insomnia, and anxiety. According to the patient, he received this wound 5 days ago, when he accidentally stepped on a glass shard, while on the beach. He requested no medical assistance. Objectively the patient’s general condition is satisfactory, pulse is 75/min., blood pressure is 130/80 mm Hg, temperature is 36.9oC. On the plantar surface of his foot there is a wound 1.5 cm long and up to 3 cm deep. The wound edges are moderately hyperemic, no discharge from the wound is observed. What disease can be suspected in this patient?

Explanation

 

The patient presents with a twitching sensation in the wound on his left foot and all other related issues began after the injury to the foot. The wound according to the presentation is infected by clostridium tetani; the causative agent of tetanus (lock jaw). This pathology is characterised by painful muscle contractions  and also affects the nervous system.

Diphtheria is an acute infectious disease caused by antigenic strains of Corynebacterium diphtheriae with predominantly airborne way of transmission, which is characterised by fibrinous inflammation in the place of bacterial invasion, development of intoxication syndrome and complications in cardiovascular, nervous and urinary systems.

187. A 45-year-old woman has been suffering from rheumatoid arthritis for 10 years and takes methotrexate twice a week. What statement regarding vaccination against pneumococci (23-valent vaccine) would conform to the recommendations for the management of rheumatoid arthritis issued by the European League Against Rheumatism in 2010?

Explanation

188. A 65-year-old woman was diagnosed with the following: chronic rheumatic heart disease, I degree of rheumatic activity; combined mitral heart disease with prevalence of III degree stenosis; heart failure IIA with retained left ventricular ejection fraction, functional class III (NYHA). What tactics of vaccination against respiratory infections should be chosen to provide secondary prevention of exacerbations and to avoid heart failure decompensation in this patient?

Explanation

 

Vaccine-preventable diseases can increase the risk of cardiovascular complications. These infections are especially important in patients with heart disease: influenza; pneumococcal; tetanus, diphtheria, and acellular pertussis (Tdap); and zoster. Patients with heart diseases are advised to take a flu shot annually. It is also recommended that patients 65 and above with CVD should receive 1 dose of the 13-valent pneumococcal conjugate vaccine (PCV13) if they did not previously receive it. Another dose of PPSV23 should be administered at least 1 year after PCV13 and at least 5 years after the first dose of PPSV23.

189. A 69-year-old woman was diagnosed with the following: ischemic heart disease; stable exertional angina pectoris, FC III; heart failure IIA with retained left ventricular ejection fraction, functional class III (NYHA). What vaccine should be chosen for influenza prevention and to avoid destabilization of the patient’s condition?

Explanation

 

Three types of influenza vaccine are widely available: inactivated influenza vaccines (IIV), recombinant influenza vaccine and live attenuated influenza vaccines (LAIV). IIV is approved for use in persons 6 months and older, including pregnant women and persons with chronic medical conditions.  One dose, injected into the deltoid thigh or muscle, is recommended. LAIV is approved for use only in persons aged 2–49 years who do not have underlying medical conditions. The vaccine should, however, not be administered to pregnant women. This patient is 69 years with an ischemic heart disease; The type of vaccine administered is not of importance.

190. A 20-year-old student was brought to the first-aid center. He has a closed fracture of the left forearm and a contused lacerated wound on his left shin. After the patient received initial wound management, he presented the documents confirming that he has received all the necessary preventive vaccination as scheduled. What should the doctor do to prevent tetanus in this patient?

Explanation

 

Note that The patient presented the document that confirms he received all the necessary vaccination at the right time ( which definitely includes a tetanus vaccine). In this case, the patient should be carefully monitored for the following few days in order to observe if any signs or changes that indicate a tetanus infection  occur.

191. A 60-year-old man presents with subcompensated viral liver cirrhosis (HCV), Child-Pugh class B. What tactics should be chosen regarding the vaccination against influenza in this case?

Explanation

Child Pugh classification is used to estimate the severity of liver diseases  according to the degree of ascites, the serum concentrations of bilirubin and albumin, the prothrombin time, and the degree of encephalopathy. A total Child-Turcotte-Pugh score of 5 to 6 is considered Child-Pugh class A (well-compensated disease); 7 to 9 is class B (significant functional compromise); and 10 to 15 is class C (decompensated disease). Two types of influenza vaccine are widely available: inactivated influenza vaccines (IIV) and live attenuated influenza vaccines (LAIV). IIV is approved for use in persons 6 months and older, including pregnant women and persons with chronic medical conditions. A flu vaccine is needed every season for two reasons. First, a person's immune protection from vaccination declines over time, so an annual vaccine is needed for optimal protection.
192. A 17-year-old girl has made an appointment with the doctor. She plans to begin her sex life. No signs of gynecological pathology were detected. In the family history there was a case of cervical cancer that occurred to the patient’s grandmother. The patient was consulted about the maintenance of her reproductive health. What recommendation will be the most helpful for prevention of invasive cervical cancer?

Explanation

Notice that from family history, there was a case of cervical cancer. Cervical cancer occurs in the cells of the cervix- the lower parts of the uterus that connects to the vagina. This malignancy is mostly caused by an infection by Human papilloma virus ( HPV 16 & 18 ). Since there is a family history, the chances of occurrence is very high therefore, vaccination against HPV is the best possible preventive/prophylactic measure.

 

HPV vaccines eg. Gardasil are recombinant vaccines administered as prophylaxis against Genital warts, cervical cancers; they target HPV strains 6, 11, 16 and 18.

193. A 6-month-old infant is not vaccinated. The physician recommends a DPT (diphtheria, pertussis, tetanus) vaccination but the mother is absolutely against this procedure. Choose the most substantial argument in favor of vaccination:

Explanation

 

There are a variety of reasons behind vaccine opposition. Some people have to forgo different vaccinations due to a high risk of potential allergic reactions, religious beliefs, mistrust of pharmaceutical companies, post vaccination complications such as autism. As a medical personnel, it is important to explain to these individuals the risk of staying unvaccinated stressing on the fact that these individuals risk exposure and stay unprotected to a wide range of diseases which in a long run will cause more damages (lethal consequences).

194. In autumn a 45-year-old man was recommended an elective surgery for coronary artery bypass grafting due to multivessel coronary artery disease. The patient has never received anti-influenza vaccination. Why would the family doctor offer a scheduled yearly vaccination against influenza to this patient?

Explanation

Vaccine-preventable diseases can increase the risk of cardiovascular complications. These infections are especially important in patients with heart disease: influenza; pneumococcal; tetanus, diphtheria, and acellular pertussis (Tdap); and zoster. Patients with heart diseases are advised to take a flu shot annually. It is also recommended that patients 65 and above with CVD should receive 1 dose of the 13-valent pneumococcal conjugate vaccine (PCV13) if they did not previously receive it. Another dose of PPSV23 should be administered at least 1 year after PCV13 and at least 5 years after the first dose of PPSV23.
195. A 28-year-old woman has made an appointment with the family doctor to receive vaccination against influenza. However, having collected the patient’s medical history, the doctor claimed this procedure to be absolutely contraindicated for this woman. What anamnestic data is the absolute contraindication to vaccination?

Explanation

Four vaccines, including those for yellow fever, influenza, measles mumps rubella (MMR), and rabies, contain small amounts of egg protein because they're cultured either in eggs or in chick embryos. 1 This raises a potential concern for people who are allergic to egg protein.
196. A 26-year-old man complains of chills, rhinitis, dry cough, and fever up to 38oC. Examination shows him to be in a moderately severe condition; there are small pale pink non-merging spots on the skin of his back, abdomen, and extremities. Palpation reveals enlarged occipital and axillary lymph nodes. No information about vaccination history could be obtained. What is the likely etiology of this disease?

Explanation

Rubella ( by rubella virus) is an acute viral infection characterised by a short prodromal period, Exanthema ( rash) during three days and lymphadenopathy. The type of rash found is roseola and small macula-papula. Note that the rash elements do not merge ( non-merging spots) and are usually concentrated on the extensor surface of extremities, back, buttocks and outer surface of thigh.

Epstein-barr virus is  one of the causative agents of Infectious mononucleosis- a disease characterised by prolonged fever, systemic lymphadenopathy, acute tonsillitis, acute adenoiditis, hepatosplenomegaly, along with typical blood changes  such as lymphocytosis, monocytosis, presence of specific cells called atypical mononuclear cells or virocytes

Mumps infection is a systemic disease characterised by hyperthermic syndrome, symmetrical or unilateral swelling of parotid glands, epididymo-orchitis etc.

 
197. During administration of planned DPT vaccination the child suddenly developed acute anxiety, signs of pain response, dyspnea, grunting respirations, cutis marmorata, cold sweat. Objectively the child’s consciousness is disturbed, heart rate is 150/min., blood pressure is 60/40 mm Hg, heart sounds are muffled. The child was diagnosed with anaphylactic shock. What drug should be administered first?

Explanation

Anaphylactic shock is a medical emergency that may require resuscitation measures such as airway management, supplemental oxygen, large volumes of intravenous fluids, and close monitoring. Administration of epinephrine is the treatment of choice with antihistamines and steroids often used as adjuncts. It is recommended that an epinephrine solution be given intramuscularly into the mid anterolateral thigh as soon as the diagnosis is suspected. The injection may be repeated every 5 to 15 minutes if there is insufficient response. A second dose is needed in 16-35% of episodes. with more than two doses rarely required. The intramuscular route is preferred over subcutaneous administration because the latter may have delayed absorption. Minor adverse effects from epinephrine include tremors, anxiety, headaches, and palpitations.

 
198. Among first-year schoolchildren there was a case of measles registered. A 7-year-old boy from the same group was not vaccinated against measles due to refusal of his parents. His clinical history has no cases of measles in the past and is not contraindicatory to immunobiological agents. Choose the most rational tactics of measles prevention in this schoolboy:

Explanation

 

The boy has not been vaccinated therefore, the most rational decision will be to vaccinate him after convincing his parents to carry out the procedure, MMR ( measles, mumps and rubella) vaccine is a combined vaccine usually administered at the age of 12 months.

199. A 40-year-old man developed fever up to 37.5*C and macular rash 10 days after the first dose of MMR (Measles-Mumps-Rubella) vaccine was administered. The vaccination was considered necessary as there was a measles outbreak in the city and the patient had not received MMR vaccination in his childhood. Is revaccination with MMR vaccine possible?

Explanation

Side effects from MMR vaccines that can occur 7 to 10 days after vaccination include:

  • fever (can be more than 39.4 °C), lasting two to three days

  • faint red rash (not infectious)

  • head cold, runny nose, cough or puffy eyes

  • drowsiness or tiredness

  • swelling of salivary glands

  • localised pain, redness and swelling at the injection site.

2 doses of the MMR vaccine provide the best protection against measles, mumps and rubella. The above listed side effects are expected in a small percentage of the population. Once the first dose is given, if any of the above listed side effects should occur, it is not a contraindication for the second dose as these side effects are expected. 

So, it is possible, as the patient in question, only had low grade fever and a macular rash.

However, it is important to note that uncommon and rare side effects like serious allergic reaction (anaphylaxis) might require a course of glucocorticoids treatment or given together with antihistamines. Thrombocytopenia, which is bleeding caused by insufficient blood platelets might be a contraindication for a revaccination. But that is not the case in this question.
200. A 26-year-old man is undergoing a regular check-up. One year ago he had a case of tonsillar diphtheria complicated with myocarditis. Presently his condition is satisfactory, no signs of cardiovascular failure; ECG shows firstdegree atrioventricular block. What vaccine was administered to this man according to his age?

Explanation

This patient had Diphtheria a year ago but currently, his condition is satisfactory. The question wants to know which vaccine was given to him a year ago that helped in managing the disease.

D- Diphtheria; T- Tetanus; P- Pertussis (Whooping cough); aP- acellular pertussis vaccine.

 

It is recommended that diphtheria, tetanus, and acellular pertussis vaccination be administered across the lifespan. Children younger than 7 years of age receive DTaP or DT, while older children and adults receive Td. Therefore, in this patient who is older than 7 years (26 year old man), he was given a diphtheria and Tetanus Vaccine.

 
  • Give infants and children 5 doses of DTaP. Give one dose at each of these ages: 2 months, 4 months, 6 months, 15 through 18 months, and 4 through 6 years. 

  • Give adolescents a single dose of Tdap, preferably at 11 to 12 years of age.

  • Give pregnant women a single dose of Tdap during every pregnancy, preferably during the early part of gestational weeks 27 through 36. 

  • Give adults who have never received Tdap a single dose of Tdap. This can be given at any time, regardless of when they last got Td. This should be followed by either a Td booster every 10 years.